Hand & Extremities Flashcards

1
Q

A 57-year-old woman presents with inability to extend the right thumb at the interphalangeal joint for the
past 3 months. The patient reports a dull aching of the right wrist for several weeks before the sudden
loss of thumb function. On physical examination, she cannot lift her right thumb when her palm is placed
on a flat surface. Seven months ago, she was treated in a cast for a non-displaced right distal radius
fracture; the fracture healed uneventfully. Which of the following is the most appropriate treatment to
restore thumb function in this patient?
A) Arthrodesis of the interphalangeal joint
B) Lengthening of the flexor pollicis longus
C) Posterior interosseous neurolysis
D) Primary repair
E) Tendon transfer

A

The correct response is Option E.
This patient has sustained a rupture of the extensor pollicis longus (EPL) tendon. Tendon transfer, using the extensor indicis proprius (EIP) tendon, is the most appropriate way to restore function to the
thumb. Although head-to-head comparison studies do not exist, the other options listed would not be
expected to restore the patient’s function, based on the clinical scenario presented.
EPL rupture is not uncommon after a distal radius fracture. Although it may not be intuitive, EPL ruptures
are actually more common following non-displaced fractures than displaced fractures. The cause of
rupture is thought to be ischemic damage leading to attritional rupture of the tendon within the tight fibro-osseous tunnel where the EPL routes around the dorsal tubercle of the radius (Lister tubercle). Because
this is an attritional rupture, primary tendon repair is usually not feasible, especially not 3 months after the
rupture has occurred.
Tendon grafting is possible but has the disadvantage of requiring two separate tenorrhaphies and a graft donor site. Both tendon transfer and/or tendon grafting may be done under local anesthesia to possibly
help set tension more accurately. An interphalangeal (IP) joint arthrodesis might help stabilize an
unstable IP joint if no tendon reconstruction is possible. However, this procedure is not indicated in most
EPL ruptures as it does not restore the missing function(s).
Flexor tendon lengthening is not indicated; this patient does not have a tight or contracted flexor
tendon. Posterior interosseous nerve (PIN) neurolysis may be indicated in cases of PIN
compression (gradual or insidious onset, pain in forearm, weakness of multiple muscles), but the patient
described in this scenario does not have these complaints.

How well did you know this?
1
Not at all
2
3
4
5
Perfectly
2
Q

A 26-year-old man sustains a laceration to the left palm with a hunting knife. Injury to the flexor digitorum
superficialis (FDS) of the ring finger is suspected. Which of the following maneuvers performed on this
patient is the most reliable method to examine FDS tendon function?
A) Flexion of the distal interphalangeal (DIP) joint with the other fingers held in extension
B) Flexion of the DIP joint with the ring finger metacarpophalangeal (MCP) and proximal interphalangeal (PIP) joints in
extension
C) Flexion of the DIP joint with the ring finger MCP joint and PIP joints in flexion
D) Flexion of the PIP joint with the other fingers held in extension
E) Flexion of the PIP joint with the ring finger MCP joint in extension

A

The correct response is Option D.
The most reliable method for examining flexor digitorum superficialis (FDS) tendon function is with flexion
of the proximal interphalangeal (PIP) joint while the other fingers are being held in extension. Each finger
contains two different flexor tendons, FDS and flexor digitorum profundus (FDP). The FDS inserts into
the middle phalanx and primarily flexes the finger at the PIP joint. The FDP inserts into the distal
phalanx and can flex both the PIP and distal interphalangeal (DIP) joints.
Flexion at the DIP joint is performed by the FDP tendon, so this tendon can be tested by stabilizing the
metacarpophalangeal (MCP) and PIP joints in extension, while observing for DIP flexion. Flexion of the
PIP joint can arise from both FDS and FDP tendon functions, regardless of whether the MCP joint is in
extension or flexion.
Testing for FDS function requires eliminating the contribution of the FDP tendon in order to isolate the
FDS. This can be done by stabilizing the other fingers in extension. The FDP has a common muscle
belly, so holding the other fingers straight will prevent the FDP from firing. The resultant flexion of the PIP
joint will be due to the FDS tendon alone. The FDS tendon has independent muscle bellies. Increased
flexion of the DIP joint when the MCP and PIP joints are in flexion is seen in cases of intrinsic tightness.

How well did you know this?
1
Not at all
2
3
4
5
Perfectly
3
Q

A 17-year-old boy presents with a mass of the left forearm that has been enlarging over the past 3
months. A photograph is shown. Biopsy of the mass is consistent with spindle cell sarcoma. MRI shows a
well-encapsulated mass that does not infiltrate neurovascular structures. CT scan of the chest shows no
evidence of metastatic lesions. Which of the following is the most appropriate next step in management?
A) Amputation at the midforearm
B) Chemotherapy only
C) Excision and chemotherapy
D) Excision and radiation therapy
E) Radiation therapy only

A

Feedback
The correct response is Option D.
Soft tissue sarcomas are rare malignant tumors representing less than 1% of all malignancies, with only
25% occurring in the upper extremity. The diagnostic workup generally includes a biopsy, magnetic
resonance imaging scan to assess the extent of the tumor and its relationship to adjacent structures, and
a computed tomography scan of the chest. This combination allows for appropriate clinical staging of the
patient. The role of sentinel lymph node biopsy in the workup of soft tissue sarcomas is controversial.
Treatment consists of wide excision, primary reconstruction, and radiation therapy (adjuvant or
neoadjuvant). The tumor must be completely removed with a cuff of normal tissue (at least 1 cm);
violation of the tumor decreases 5-year survival from 87 to 47%. More than 90% of extremity sarcomas
can be managed with a limb-sparing resection.
Chemotherapy is rarely indicated and is never used as the only treatment for soft tissue sarcoma.
Primary amputation is considered when the tumor infiltrates major neurovascular structures and resection
would result in the sacrifice of more than one major peripheral nerve. Additional indications for
amputation include involvement of the interosseous membrane, advanced disease with extensive loss of
functional tissues, and severe comorbidities limiting reconstructive options.

How well did you know this?
1
Not at all
2
3
4
5
Perfectly
4
Q

A 32-year-old man presents to the emergency department for evaluation of a laceration of the right wrist sustained when he punched a glass window 1 hour ago. Physical examination shows a 2-cm transverse
laceration of the volar ulnar wrist crease. Wound exploration shows complete laceration of the ulnar
nerve. On physical examination of motor function, LOSS of which of the following functions is most likely
in this patient?
A) Adduction of the thumb carpometacarpal joint
B) Extension of the metacarpophalangeal joint of the ring and small fingers
C) Extension of the thumb interphalangeal joint
D) Flexion of the interphalangeal joint of the index and middle fingers
E) Flexion of the interphalangeal joint of the ring and small fingers

A

The correct response is Option A.
The ulnar nerve is the terminal branch of the medial cord of the brachial plexus. It enters the forearm
between the two heads of the flexor carpi ulnaris (FCU). In the forearm, the ulnar nerve innervates the
FCU and flexor digitorum profundus of the small and ring fingers. It courses distally under the FCU to
enter Guyon’s canal at the wrist. The dorsal cutaneous nerve, which gives sensation to the dorsoulnar
hand, arises approximately 5 to 7 cm proximal to the ulnar styloid. In Guyon’s canal, the ulnar nerve splits
into a deep motor and a superficial sensory branch. The deep motor branch innervates the hypothenar
muscles (abductor digiti minimi, opponens digiti minimi, and flexor digiti minimi), as well as the lumbricals
to the ring/small fingers, dorsal and palmar interossei, flexor pollicis brevis (deep head), palmaris brevis and adductor pollicis. The superficial sensory branch in the palm innervates the small finger and the ulnar
aspect of the ring finger.
Adduction of the thumb is controlled through activation of the ulnar-innervated adductor pollicis muscle.
In the small and ring fingers, extension of the metacarpophalangeal (MCP) joint is performed through
activation of the radially innervated extensor digitorum communis and extensor digiti minimi muscles. In
the index, middle, ring, and small fingers, flexion of the proximal interphalangeal (PIP) joint is performed
through activation of the median-innervated flexor digitorum superficialis.
While flexion of the ring and small finger distal interphalangeal (DIP) joints is produced by the ulnar-nerve innervated FDP tendons to the ring and small finger, the ulnar nerve provides branches to this muscle
proximal to this patient’s injury. Extension of the thumb MCP joint is via the radial nerve innervated
extensor pollicis brevis muscle.

How well did you know this?
1
Not at all
2
3
4
5
Perfectly
5
Q

A 29-year-old man undergoes evaluation for nonunion of a scaphoid fracture. Reconstruction with a
vascularized osseous flap is planned, and a medial femoral condyle flap is chosen. During harvest, the
vascular pedicle for this flap runs between which of the following structures?
A) Anterior to the tensor fascia lata and posterior to the vastus lateralis
B) Anterior to the vastus medialis and anterior to the adductor tendon
C) Anterior to the vastus medialis and posterior to the rectus femoris
D) Posterior to the rectus femoris and anterior to the vastus lateralis
E) Posterior to the vastus medialis and anterior to the adductor tendon

A

The correct response is Option E.
The medial femoral condyle osseous free flap has become a useful option for reconstruction of bony
defects in the extremities, particularly of the scaphoid waist and proximal pole. The vascular supply to
this flap is from the descending geniculate artery in the distal medial aspect of the thigh. To explore and
identify the pedicle for this flap, the vastus medialis is reflected anteriorly, and the adductor tendon is
found posterior to the vessels. The rectus femoris is located anterior to the dissection for this flap.

How well did you know this?
1
Not at all
2
3
4
5
Perfectly
6
Q

A 2-year-old female infant presents with a 3 × 3-cm red, firm, raised mass on the dorsum of the right
hand. Physical examination shows a red mass with a smooth surface, and a rim of decreased
pigmentation. The patient’s mother reports that the mass has not changed in size, appearance, or
coloration since birth. An ultrasound and MRI at age 3 months showed a well-defined, homogenous mass
with high-blood flow characteristics. Which of the following is the most likely diagnosis in this patient?
A) Dermatofibrosarcoma protuberans
B) Infantile hemangioma
C) Lymphatic malformation
D) Non-involuting congenital hemangioma (NICH)
E) Venous malformation

A

The correct response is Option D.
Non-involuting congenital hemangioma (NICH) is a rare form of hemangioma that is present at birth, is
stable in size over time (ie, does not involute), and often has a white-grey rim. It is histologically and
radiographically similar to infantile hemangioma except that it stains negative for glucose transporter
protein 1 (GLUT1).
Venous malformation and lymphatic malformation are low-flow and do not fit the clinical description: they
are typically darker in color, amorphous in form, and are compressible. Dermatofibrosarcoma
protuberans is rare in children, begins more like a flat scar, and grows over time. It is not a high-flow
lesion.

How well did you know this?
1
Not at all
2
3
4
5
Perfectly
7
Q

A 32-year-old woman presents to the emergency department with a transverse laceration over the volar
nondominant small finger. Physical examination shows full active flexion and extension but pain on
resisted flexion. Exploration shows a 20% flexor profundus laceration. Which of the following are the
most appropriate joint positions for splinting this patient’s wrist, metacarpophalangeal (MCP) joints,
proximal interphalangeal (PIP) joints, and distal interphalangeal (DIP) joints?
A) Wrist extension, MCP joint extension, PIP joint extension, DIP joint extension
B) Wrist extension, MCP joint flexion, PIP joint extension, DIP joint extension
C) Wrist extension, MCP joint flexion, PIP joint flexion, DIP joint extension
D) Wrist flexion, MCP joint extension, PIP joint extension, DIP joint extension
E) Wrist flexion, MCP joint flexion, PIP joint flexion, DIP joint flexion

A

The correct response is Option B.
The correct position in which to immobilize the hand is intrinsic plus position. This is also known as the
safe position that helps to prevent joint stiffness and joint contractures. This goal is to have the collateral
ligaments of the wrist, metacarpophalangeal (MCP) joint, proximal interphalangeal (PIP) joint, and distal
interphalangeal (DIP) joint at full tension. The wrist is placed between 0 to 30 degrees of extension, the
metacarpophalangeal (MCP) joints in 70 to 90 degrees of flexion, and both the PIP and DIP joints into full
extension. Since the patient has only a partial-thickness flexor tendon injury, there is no need to consider
flexion other than at the MCP joint.

How well did you know this?
1
Not at all
2
3
4
5
Perfectly
8
Q

A 37-year-old man who works as a laborer sustains a saw injury to the non-dominant left hand. X-ray
studies are shown. Which of the following is the most appropriate functional option for reconstruction of
this thumb defect?
A) Metacarpal lengthening
B) Osteoplastic reconstruction
C) Pollicization
D) Prosthesis
E) Toe transfer

A

The correct response is Option C.
The x-ray study shows a carpometacarpal-level amputation of the thumb. Toe transfer, osteoplastic
reconstruction, and metacarpal lengthening require part or most of the thumb metacarpal to be present. A
thumb prosthesis would have limited functionality and be insensate.
Pollicization would potentially allow a sensate, functional index finger to accomplish some of the functions
of the thumb. It is, however, not without its drawbacks because it is technically demanding and has a high
likelihood of requiring secondary procedures. For a carpometacarpal-level amputation, pollicization
provides the best option for function.

How well did you know this?
1
Not at all
2
3
4
5
Perfectly
9
Q

A 56-year-old woman with a traumatic defect of the upper third of the tibia undergoes open reduction and
internal fixation with tibial nail. Soft tissue coverage with a gastrocnemius flap is planned. Which of the
following arteries provides the dominant blood supply for this flap?
A) Anterior tibial
B) Peroneal
C) Popliteal
D) Posterior tibial
E) Sural

A

The correct response is Option E.
Each head of the gastrocnemius muscle is supplied by the sural artery: either the medial sural or lateral
sural artery for medial and lateral gastrocnemius, respectively. The arteries arise from the popliteal artery
about 3-4 cm above the head of the fibula and enter the medial and lateral heads of the gastrocnemius at
about the level of the head of the fibula. The flap can be rotated to cover soft-tissue defects of the
anterior distal aspect of the knee. The flap ranges from 5 to 9 cm in width and from 13 to 20 cm in length.
It provides a vascular bed for a skin graft and improves the delivery of oxygen and systemic antibiotics.
The other listed arteries do not supply the gastrocnemius muscles.

How well did you know this?
1
Not at all
2
3
4
5
Perfectly
10
Q

A 9-month-old boy presents for evaluation of unilateral preaxial polydactyly. X-ray studies show
triphalangism of the accessory thumb. Which of the following is the most likely Wassel classification in
this patient?
A) Type II
B) Type III
C) Type IV
D) Type V
E) Type VII

A

The correct response is Option E.
Preaxial polydactyly describes patients with thumb duplication. In 1969, Wassel (as the fellow of Adrian
Flatt) described a categorization system for radial polydactyly corresponding to the level of skeletal
duplication. There have been many subsequent modifications of this classification system, but this
remains the most commonly used. Type IV (duplicated proximal and distal phalanges) is the most
common duplication, followed by Type II (duplicated distal phalanx). A Type VII duplication involves a
triphalangeal thumb and is the only deformity in which there is triphalangism.

How well did you know this?
1
Not at all
2
3
4
5
Perfectly
11
Q

A 65-year-old woman presents with severe osteoarthritis of the proximal interphalangeal (PIP) joint of the
nondominant left middle finger. Medical history includes chronic pain and an angular deformity of the
joint. Range of motion of the PIP joint is 30 to 60 degrees. Silicone implant arthroplasty is planned. Which
of the following is the principle benefit of this procedure?
A) Correction of angular deformity
B) Improved cosmesis
C) Improved range of motion
D) Increased grip strength
E) Pain relief

A

The correct response is Option E.
Expected outcomes for small joint implant arthroplasty are pain relief with similar range of motion to
preoperative values. The procedure involves excision of the arthritic proximal phalanx head and middle
phalanx base and replacement with a silicone stemmed implant. The implant acts as a spacer for
development of a scar capsule.
Although angular deformity is corrected with this procedure, and many patients report satisfaction with
the improved appearance of the alignment of the finger, the primary goal of the procedure is pain relief
from underlying arthritis. Outcome studies have not demonstrated improved grip strength or range of
motion. Long-term outcome studies show 90% implant survival at 10 years, high patient satisfaction, and
a low revision rate despite a relatively high incidence of implant fracture or deformity over time.

How well did you know this?
1
Not at all
2
3
4
5
Perfectly
12
Q

A 6-year-old boy presents with a supracondylar fracture sustained during a fall on an outstretched hand.
A splint with the elbow flexed less than 90 degrees is placed. The patient is screaming in pain.
Examination shows the affected hand has a 3-second capillary refill. Which of the following is the most
appropriate next step in management?
A) Closed reduction
B) Continued observation and application of ice packs
C) Elevation of the arm
D) Exploration of the brachial artery
E) Replacement of the current splint with an elbow extension splint

A

The correct response is Option A.
Supracondylar fractures are one of the most common traumatic fractures seen in children. It occurs most
commonly in children 5 to 7 years of age with similar male and female incidence. The mechanism is
usually from a fall onto an outstretched hand. The fracture can lead to severe forearm edema, then
ischemia leading to Volkmann’s contracture.
Immobilization would be long arm casting with the elbow flexed at less than 90 degrees. Arm elevation
would decrease tissue perfusion and would therefore be contraindicated. Immediate bedside closed
reduction by gentle traction and elbow flexion to 20 to 40 degrees would be indicated in this case as a
next step. If the closed reduction is unsuccessful or ischemia persists after reduction or re operative closed reduction with percutaneous pinning is required. Pins are placed to prevent recurrence.
Brachial artery exploration could be required if ischemia has not resolved even after successful reduction,
but not initially.

How well did you know this?
1
Not at all
2
3
4
5
Perfectly
13
Q

A 40-year-old woman presents with small, non-healing ulcers of the right index and middle fingertips.
Medical history includes limited scleroderma diagnosed 5 years ago, chronic pain, and color changes of
the fingers in cold temperatures. The patient’s symptoms have not improved with administration of
nifedipine. Angiography shows diffuse vascular narrowing without any focal lesions. Which of the
following is the most appropriate intervention for pain relief and ulcer healing in this patient?
A) Cervical sympathectomy
B) Continuous brachial plexus blockade
C) Digital bypass
D) Onabotulinum toxin A
E) Stellate ganglion block

A

The correct response is Option D.
This patient has Raynaud’s phenomenon associated with scleroderma. The pathophysiology of
Raynaud’s is thought to be related to sympathetic hyperactivity, elevated plasma endothelin, increased
peripheral alpha-2 receptors, and possibly abnormal platelet and red cell function. Botulinum toxin type A
has been shown to improve digital perfusion on laser Doppler, decrease pain, and result in ulcer healing.
In a series of 33 patients injected with 50 to 100 U of onabotulinum toxin A, all patients had ulcer healing
by 60 days postinjection. Pain relief typically occurred within 5 to 10 minutes of injection and complication
rates were low and limited to injection site reactions. A prospective, randomized, placebo-controlled trial
showed patients with limited scleroderma and shorter duration of disease had the best response to
onabotulinum toxin A.
Stellate ganglion blocks have been shown to have only variable success for Raynaud’s with only short-term symptom relief and no effect on ulcer healing. Stellate blocks may not disrupt all sympathetic input
to the extremity. Brachial plexus blocks may help with perfusion temporarily but are advocated mainly in
patients undergoing microvascular surgery. Their use is not recommended in this setting. Surgical
bypass to the superficial palmar arch has been shown to increase blood flow to the hand and improve
ulcer healing. However, bypass to the digital vessels would not be indicated as the distal target vessels
are often diminutive without adequate flow.

How well did you know this?
1
Not at all
2
3
4
5
Perfectly
14
Q

A 40-year-old man presents to the emergency department because of severe pain after sustaining a
crush injury to the left lower extremity from a forklift. On physical examination, the lower leg is tense and
swollen circumferentially. Sensation to the foot is diminished. Distal pulses are palpable. X-ray study
does not show any fractures. Which of the following is the most appropriate next step in management?
A) Ace wrap compression
B) CT angiography
C) Emergent fasciotomy
D) MRI
E) Observation and leg elevation

A

The correct response is Option C.
The patient displays the signs and symptoms of acute compartment syndrome, a surgical emergency
requiring emergent fasciotomy. Acute compartment syndrome requires prompt diagnosis and expeditious
treatment in order to minimize morbidity.
Compartment syndrome can occur following a substantial soft tissue crush injury, even in the absence of
a fracture, such as in this clinical scenario. Severe pain is usually the presenting complaint. It may be out
of proportion to the injury and unresponsive to analgesics. The presence of paresthesias can signify
nerve hypoxia from elevated compartment pressures. Pallor, paralysis, and pulselessness are very late
signs. Nerve and muscle do not tolerate long periods of ischemia and may undergo irreversible damage if
surgical decompression is delayed.
Compartment syndrome is primarily a clinical diagnosis, but measurement of compartment pressures can
provide additional information especially if the diagnosis of compartment syndrome is less obvious. If
compartment pressures are greater than 30 mmHg or if the differential pressure (difference between
diastolic blood pressure and compartment pressure) is less than 30 mmHg, then fasciotomy is
recommended.
Observation and leg elevation would not be appropriate management in the setting of acute compartment
syndrome. CT angiography would not be indicated in this case, where there is a low suspicion of vascular
injury. MRI has been used in the diagnosis of chronic exertional compartment syndrome but has little value in the setting of acute trauma.

How well did you know this?
1
Not at all
2
3
4
5
Perfectly
15
Q

A 19-year-old right-hand-dominant man presents with an injury to the right ring finger sustained when he
hung on the basketball rim after slam dunking a basketball 1.5 weeks ago. Photographs are shown. X-ray
studies are unremarkable. Which of the following is the most appropriate next step in management?
Figure 15-1 Figure 15-2
A) Fusion of distal interphalangeal joint in 20 degrees of flexion
B) Primary repair
C) Staged tendon reconstruction
D) Tenodesis
E) Observation

A

The correct response is Option B.

This patient has an avulsion of the flexor digitorum profundus (FDP) tendon from its insertion at the volar
base of the distal phalanx (Zone 1), also known as a jersey finger. The ring finger is involved in 75% of
cases of this type due to its prominence during grip (longer than adjacent fingers during grip in most
patients). Leddy and Packer classify jersey finger injuries as Types 1 through 5:
Type 1: Avulsion with retraction into the palm with disruption of the vincular blood supply,
Type 2: Retraction to the level of the proximal interphalangeal (PIP) joint,
Type 3: Associated fracture fragment holds tendon at distal interphalangeal (DIP) joint level,
Type 4: Fracture with tendon avulsion from bony segment,
Type 5: Fracture, avulsion, with distal phalangeal comminution.
Since there was no bony fragment on x-ray study, this is either a Type 1 or 2 and an attempt at primary
repair up to 3 weeks after injury is almost invariably achievable. This offers the best possible outcome
versus the other options, which are either salvage or not practical. Observation is a poor alternative as
the loss of the FDP can result in DIP joint hyperextension and can adversely affect the ring finger mobility
and function. Fusion is a salvage procedure and should not be considered when repair is possible.
Staged repair could be considered in a more chronic injury but is tedious and has variable outcomes. It is
unnecessary in this patient. Tenodesis is another salvage procedure that has been described in
unrepairable injuries.

How well did you know this?
1
Not at all
2
3
4
5
Perfectly
16
Q

A 12-year-old boy presents with warts on his fingers. His parents report that the warts have persisted
despite application of a variety of topical treatments. Which of the following viruses is the most likely
cause of this patient’s condition?
A) H1N1
B) Hepatitis C
C) Herpes simplex
D) Human immunodeficiency
E) Human papilloma

A

e correct response is Option E.
The cause of warts is the human papilloma virus (HPV). There are more than 100 known types of HPV.
Types 2 and 4 are the most common cause of warts on the hands, as in this patient. People whose
occupations expose their hands to wet environments, such as meat, poultry, and fish handlers and
veterinary surgeons have higher risk of developing warts. The virus can be transmitted on shared
clothing or public spaces, such as showers, and then gain entry through breaks in the skin. The virus is
then able to alter the squamous epithelium to produce a cauliflower-like growth. Warts can be present in
single lesions or multiple lesions. They are often painless and are usually not cancerous but can be a
source of embarrassment for the patient. Warts are generally self-limited and will resolve spontaneously
within months or years. Spontaneous resolution appears to occur in 50% of children within 1 year and
90% within 5 years.
There are many treatments for warts. Home remedies include topical salicylic acid, duct tape, and cold
treatments. Clinical treatments have shown topical acids and cryotherapy to be equally effective.
Intralesional injections and topical antivirals, as well as immunologic stimulators of interferon, have been
tried with some success, including purified candida; measles, mumps, and rubella; and tuberculin (PPD)
protein. Chemical ablation with silver nitrate has been shown to resolve almost half of warts a month after
a 9-day treatment protocol. Mechanical removal can be performed with direct excision with good
success. Pulsed dye and CO2 lasers have been successfully used to ablate warts but can be painful,
expensive, and leave scars. Periungal warts can be particularly difficult to treat topically and may require
a more invasive treatment method. A patient’s ability to eradicate a wart can be reduced by a
compromised immune system. Multiple progressive warts in immunosuppressed patients may need to be
biopsied as these warts may transform into squamous cell carcinoma.
Human immunodeficiency virus (HIV), H1N1 , hepatitis C, and herpes simplex are all viruses that affect
humans but do not cause lesions on the fingers. Herpes simplex virus is the cause of cold sores around
the mouth and genital herpes. Hepatitis C is a virus that causes inflammation of the liver. HIV decreases
the effectiveness of the immune system. H1N1 is an influenza virus and known cause of the “swine flu” outbreak in 2009.

How well did you know this?
1
Not at all
2
3
4
5
Perfectly
17
Q

A 41-year-old woman who is a dentist comes to the office because she has had intense burning and
pruritus of the dominant index finger for the past 6 days. The patient reports a small vesicular rash on the
finger that has since progressed to form a small blister. She has had intermittent fever and malaise.
Which of the following is the most appropriate management?
A) Incision and drainage of the bullae
B) Intravenous administration of cefazolin
C) Oral administration of acyclovir
D) Topical application of silver sulfadiazine
E) Observation only

A

The correct response is Option E.

The patient has a history and physical findings consistent with herpetic whitlow. This is a viral infection
caused by herpes simplex virus and is more common in medical and dental personnel. Tzank smear or
antibody titers can confirm a diagnosis but are unnecessary in the management of this patient. Treatment is primarily nonoperative and involves observation, as the course of the illness is self limiting with resolution in 1 to 3 weeks. Intravenous antibiotics would not treat this viral infection. Incision and drainage is unnecessary and may lead to a bacterial superinfection or systemic dissemination of herpes simplex virus. Acyclovir or valacyclovir may shorten the duration of symptoms, but must be started within 2 to 3 days of onset. Topical application of an antimicrobial would provide no benefit in this case. Surgical drainage of the bullae should not be performed because it may increase the risk of spreading the herpes virus and may also lead to bacterial superinfection.

How well did you know this?
1
Not at all
2
3
4
5
Perfectly
18
Q

A 53-year-old African American man is referred for evaluation of a 3-mm-wide pigmented streak of the
left index fingernail. A full-thickness biopsy of the nail matrix confirms the diagnosis of melanoma. Which
of the following factors is most pertinent in determining prognosis and 5-year survival in this patient?
A) Mitotic rate
B) Tumor free resection margins
C) Tumor location
D) Tumor stage
E) Width of the lesion

A

The correct response is Option D.
This patient has acral lentiginous melanoma (ALM) based on the clinical description of a dark-skinned
man with a tumor on the fingernail presenting as a pigmented streak. The prognosis for ALM is typically
worse than other melanoma subtypes. The poor survival rate of these patients may be due to a delay in
diagnosis. As in other melanoma subtypes, tumor thickness is the most important prognostic indicator. Overall, 5-year survival for ALM is 80 versus 91% for all melanomas. Acral lentiginous
melanoma is the least common subtype of melanoma, however, it makes up the highest percentage of
cutaneous melanomas in dark-skinned patients. ALM is predominantly found on the palms, soles, and
nail beds. This is in contrast with other melanoma subtypes that typically occur in sun-exposed areas.
However, the location of the tumor does not directly influence the prognosis. It is more correlated to
diagnosis at a later stage.
Mitotic rate and other pathologic characteristics such as microscopic ulceration, lymphatic, or nerve
involvement can upstage the tumor. However, it is not the primary determinant of tumor stage, and
mitotic rate is no longer considered as part of staging in early melanomas.
Achieving adequate resection margins in ALM may be difficult, especially in tumors involving the nail
unit. Amputation at the next most proximal joint is often recommended. Regardless of the status of the
margins, prognosis is still determined by depth at diagnosis. Thicker tumors have a higher incidence of
nodal involvement and metastatic disease. In later stage disease, surgical resection of the primary tumor
is for diagnostic purposes, local control, and occasionally palliative care.
Pigmented lesions of the nails greater than 2 mm have a higher likelihood of being invasive melanoma,
but width of the lesion is not involved in tumor staging or prognosis.

How well did you know this?
1
Not at all
2
3
4
5
Perfectly
19
Q

In a transhumeral amputee, targeted muscle reinnervation can be utilized to improve control in a
myoelectric prosthesis. Which of the following nerve transfers can be performed to provide intuitive
prosthetic control for hand closure?
A) Median nerve to short head of biceps
B) Musculocutaneous nerve to long head of biceps
C) Radial nerve to lateral head of triceps
D) Radial nerve to long head of triceps
E) Ulnar nerve to lateral head of triceps

A

The correct response is Option A.
Targeted muscle reinnervation (TMR) utilizes a set of nerve transfers in order to allow intuitive prosthetic
control for upper extremity amputees. Functioning nerves that no longer have their distal muscle target
can be transferred to intact proximal muscles and generate a novel electrical signal that can be picked up
by a myoelectric prosthesis. Another benefit of TMR is the potential to prevent or treat painful neuromas.
In the case of a transhumeral amputee, elbow flexion myoelectric prosthetic control is maintained by
preserving musculocutaneous innervation to the long head of the biceps muscle. The distal remnant of
the median nerve is transferred to the motor nerve of the biceps short head to create a signal for
prosthesis hand closure. Elbow extension signals are maintained with radial innervation of the long head
of the triceps. Signals for prosthesis hand opening are created with transfer of the distal radial nerve to
the motor nerve of the triceps lateral head.

How well did you know this?
1
Not at all
2
3
4
5
Perfectly
20
Q

A 2-year-old male infant presents with a congenital deformity of the ring finger. A photograph and x-ray
study are shown. The patient’s parents report that the condition limits his ability to make a full fist but is
otherwise painless. Which of the following is the most likely diagnosis in this patient?
A) Amniotic band syndrome
B) Camptodactyly
C) Congenital trigger finger
D) Macrodactyly
E) Syndactyly

A

The correct response is Option A.
Amniotic band syndrome (ABS) has an incidence of 1/1200 to 1/15,000 births. Some congenital
anomalies have been associated with ABS including cleft palate, imperforate anus, equinovarus, and
body wall defects.
The etiology of ABS has two theories. The intrinsic defect theory endorses genetic abnormalities which
lead to mesenchymal hypoplasia and scarring. The extrinsic theory endorses amniotic tissue which
entangles fetal parts leading to constriction.
Constriction varies leading to a spectrum of clinical manifestations from skin dimpling to digital
amputation. Neurovascular injury can manifest as peripheral nerve palsy, lymphedema and arterial
insufficiency. Syndactyly is common, and acrosyndactyly is pathognomonic of ABS. The Patterson
classification system has four subtypes. The first is a simple constriction ring. The second has a
constriction ring that affects the digit distal to the ring, with or without lymphedema. The third consists of
constriction rings with acrosyndactyly. The fourth is characterized by amputation at any level.
Camptodactyly is defined as a painless and progressive non-traumatic contracture of the proximal
interphalangeal (PIP) joint. It affects around 1% of the population and the great majority of the cases are
extremely mild and asymptomatic. The cause of the contracture is controversial. There have been
descriptions of malformations of the superficial flexor of the fingers, lumbrical muscles, and the
transverse and oblique retinacular ligaments. There may also be alterations to the configurations of the
PIP joint.
Congenital trigger finger differs from congenital trigger thumb. Congenital trigger finger is rare and
presents more commonly in the ulnar digits with associated malformations of the superficial and deep
flexors. It presents with sporadic locking. Release of the A1 pulley alone is not adequate, with tenoplasty
of the chiasm and partial opening of the A2 pulley generally necessary.
Syndactyly is a variable fusion between two adjacent fingers, and is one of the most common congenital
deformities, occurring in 1:2000 live births.
Classification includes:
Simple: fusion only through the skin
Complex: bone connection.
Complete: the entire commissure is involved, including the nail bed
Incomplete: the nail bed is not involved.
Complicated: involvement of vascular tissues, tendons or nerves.
It can occur separately or as a manifestation of a syndrome, such as Streeter, Apert or Poland, in which
the severity of the syndactyly is more significant.
Macrodactyly is a congenital overgrowth disorder and represents 0.9% of upper extremity congenital
anomalies. Digital enlargement involves all tissue types and maintains patterns of growth and anatomic
relationships within the affected portion of the hand.
The term “macrodactyly” is reserved for nonsyndromic, congenital enlargement of a digit or digits that
occurs in isolation without associated limb hemihypertrophy or vascular anomaly.

How well did you know this?
1
Not at all
2
3
4
5
Perfectly
21
Q

In patients with rheumatoid arthritis, the inciting event in development of a boutonniere deformity is which
of the following?
A) Central slip attenuation
B) Intrinsic tightness
C) Lateral band volar subluxation
D) Oblique retinacular ligament contracture
E) Proximal interphalangeal (PIP) joint synovitis

A

The correct response is Option E.
Rheumatoid arthritis is a chronic autoimmune disease characterized by inflammation and deterioration of the joints. Synovial proliferation is the hallmark of rheumatoid arthritis and is often seen early in the course of the disease. There is a progression to synovial pannus formation, periarticular bone
demineralization, cartilage destruction, and subchondral osseous erosions. This process is mediated by synovial infiltration of activated T lymphocytes, which promote chronic synovial inflammation.
The boutonniere deformity is extremely common in patients with rheumatoid arthritis. It is characterized by flexion of the proximal interphalangeal (PIP) joint with hyperextension of the distal
interphalangeal (DIP) joint.
The causative event of boutonniere deformity in rheumatoid arthritis is synovitis and synovial pannus
formation within the PIP joint. This causes the joint capsule to distend, resulting in attenuation of the
central slip. Central slip insufficiency results in loss of PIP joint extension and subsequent volar translocation of the lateral bands, which further accentuates the deformity by providing a flexion force across the PIP joint. Extension forces are transferred to the DIP joint. Contraction of the oblique retinacular ligament is associated with a fixed deformity.
Intrinsic tightness would cause the PIP joint to be unable to be flexed when the MP is in extension.

How well did you know this?
1
Not at all
2
3
4
5
Perfectly
22
Q

A 23-year-old man presents with painful, red swelling over the dorsum of the right middle finger
metacarpophalangeal (MCP) joint 3 days after he punched someone in the face during a bar fight. The
patient reports decreasing range of motion of the hand and inability to grasp objects. He was previously seen in the emergency department the night of the injury and x-ray studies were negative for fracture or
foreign body. Which of the following is the most likely causative organism?
A) Clostridium perfringens
B) Eikenella corrodens
C) Pasteurella multocida
D) Pseudomonas aeruginosa
E) Staphylococcus epidermidis

A

The correct response is Option B.
Eikenella corrodens is an anaerobic organism present in human oral flora and has been associated with human bite wounds. Group A Streptococcus is also a common pathogen in a fight bite injury like the one this patient has.
This patient has most likely sustained a “fight bite,” which results from tooth penetration of the
metacarpophalangeal (MCP) joint after striking someone in the mouth with a clenched fist. These injuries
can often be underappreciated, as the underlying defect in the extensor hood and joint capsule may not
be seen on examination when the fingers are extended during examination in an emergency department.
The joint can become contaminated with oral flora. Penetrating injury with high bacterial load can result in
a septic joint and lead to destruction of cartilage and osteomyelitis. Recreation of the flexed fist position
may help in lining up the structures and assist in identification of the injury. Treatment is aggressive
antibiotic therapy and surgical exploration with irrigation and debridement of the joint to remove debris.
Clostridium perfringens is a gram-negative rod associated with gas gangrene, which results in
subcutaneous crepitus and can be rapidly progressive. Pasteurella multocida is a gram-negative
anaerobic bacterium most commonly associated with cat bite infections. Pseudomonas aeruginosa is a
gram-negative rod that can be associated with diabetic wound infections. And Staphylococcus
epidermidis is a gram-positive cocci present on the skin. It has been associated with implant infections.
None of these pathogens are as likely to be present in a fight bite as Eikenella corrodens.

How well did you know this?
1
Not at all
2
3
4
5
Perfectly
23
Q

A 30-year-old man presents to the emergency department with acute left wrist pain after falling 10 feet
from a ladder. X-ray studies of the left wrist are shown. After failed closed reduction, the patient reports tingling that progresses to worsening and constant numbness of the left index and long fingers over the
course of 6 hours. Which of the following urgent interventions is most appropriate?
A) Aspiration of the wrist
B) Carpal tunnel release
C) MRI of the wrist
D) Open reduction of the scaphoid
E) Repeat closed reduction

A

The correct response is Option B.
This patient has a type IV perilunate dislocation, or a true lunate dislocation. This represents a complete
disruption of the ligamentous stabilizers about the lunate. These injuries are high energy and can be
ligamentous only (lesser arc injuries) or include fractures (greater arc) and are then termed perilunate
fracture dislocations. Mayfield et al described the stages of injury progressing from radial to ulnar in a
type IV dislocation, including injury of the scapholunate ligament, disruption of the lunocapitate joint,
injury of the lunotriquetral ligament, and dislocation of the lunate from its fossa at the radiocarpal joint
volarly into the carpal tunnel. On posteroanterior x-ray study of the wrist, there will be disruption of
Gilula’s lines. On lateral x-ray study, a “spilled teacup” sign is seen.
Closed reduction with relaxation and traction is important, as the lunate needs to be relocated to its fossa
to restore relative alignment of the wrist and to decompress the median nerve in the carpal tunnel.
Surgical intervention can then be performed for open reduction of the joints and ligament repair after
swelling has improved. However, progression in median nerve symptoms in the setting of successful or
failed closed reduction is indicative of acute carpal tunnel syndrome and necessitates urgent surgical
intervention.
Advanced imaging such as MRI is not required but may be helpful. Repeat closed reduction is likely to
fail at this time, may worsen the swelling, and is unlikely to resolve the carpal tunnel symptoms. Open
reduction of the scaphoid is not emergent, and the patient does not have a scaphoid fracture. Aspiration
of the wrist will not resolve the inciting etiology of the patient’s carpal tunnel symptoms.

How well did you know this?
1
Not at all
2
3
4
5
Perfectly
24
Q

Resistance to which of the following maneuvers is most likely present in a digit that has intrinsic
tightness?
A) Passive extension of the metacarpophalangeal (MCP) joint with the proximal interphalangeal (PIP) joint held in hyperextension
B) Passive extension of the PIP joint with the MCP joint held in hyperflexion
C) Passive flexion of the DIP joint with the PIP joint held in hyperextension
D) Passive flexion of the PIP joint with the MCP joint held in hyperextension
E) Passive flexion of the PIP joint with the MCP joint held in hyperflexion

A

The correct response is Option D.
The intrinsic muscles (dorsal/palmar interossei and lumbricals) are responsible for much of the fine motor function of the hand. Contractures of these muscles lead to a loss of the delicate and complex balance of the intrinsic and extrinsic muscles and typically results in the clinical picture of an intrinsic-plus hand. The intrinsics attach to the extensor mechanism through the lateral bands and facilitate force transmission from the muscles to the extensor mechanism on the proximal and distal phalanges. Because of their line of pull, the intrinsics are responsible for metacarpophalangeal (MCP) joint flexion and proximal interphalangeal (PIP) joint extension. The intrinsic tightness test (i.e. Bunnell test) requires one to assess passive PIP joint flexion with the MCP joint extended. This is compared with passive PIP joint flexion with
the MCP joint in flexion which assesses for extrinsic tightness. If there is a substantial increase in
resistance to PIP joint flexion with the MCP joint in extension, then the test is considered positive and
indicative of intrinsic tightness or adhesions of the lateral bands.
Trauma is the most common cause of intrinsic muscle contracture. Spasticity from an upper motor
neuron lesion (e.g. traumatic brain injury, cerebrovascular accident, cerebral palsy) may also lead to
intrinsic contracture. Arthritis may also lead to intrinsic contracture resulting from joint deviation or
dislocation.
In trauma, initial treatment is directed at edema prevention and aggressive hand therapy. Patients with spasticity from an upper motor neuron disorder are also initially managed with therapy and splinting. If these modalities are insufficient, surgical release of the intrinsic muscles or tendons (proximal or distal depending on extent of involvement) may improve posture and function. Ulnar motor neurectomy is
another option in severely affected individuals to decrease intrinsic muscle tone and improve posture and function, but is only effective in the absence of a fixed MCP joint contracture.

How well did you know this?
1
Not at all
2
3
4
5
Perfectly
25
Q

Which of the following failures in embryologic development is most likely to have caused the deformity
pictured in the photographs shown?
A) Differentiation of the zone of polarizing activity
B) Formation of the apical ectodermal ridge
C) Inhibition by en-1
D) Longitudinal formation
E) Programmed cell death

A

The correct response is Option E.
The hand plate initially forms with webbed digits. In order to have separate fingers, the interdigital tissue
must undergo programmed cell death/apoptosis. Bone morphogenetic protein (BMP) plays an essential
role in this process. A complex interplay creates failure of longitudinal formation (i.e., radial club hand).
Removal of the apical ectodermal ridge (AER) results in limb truncation. The zone of polarizing activity
(ZPA) is present in the posterior aspect of the developing upper limb and helps direct the anteroposterior
axis (radial-ulnar axis). Sonic hedgehog (SHH) is the critical signaling factor. ZPA transplantation or
excess SHH results in mirror hand deformity. The dorsoventral axis is another pathway critical for
appropriate limb development. The WNT7A signaling molecule is expressed in the developing upper limb
dorsal ectoderm, activating the LIM homeodomain, resulting in the expression of LMX1B transcription
factor from the dorsal mesenchyme, whereas the ventral ectoderm induces the expression of en-1.
These signaling factors are necessary for the formation of dorsal versus palmar structures of the hand.

How well did you know this?
1
Not at all
2
3
4
5
Perfectly
26
Q

A 35-year-old man presents for evaluation of a laceration to the lateral aspect of the right lower leg 5 cm distal to the knee that he sustained when he fell from a bicycle 2 months ago. Findings on
electromyography and nerve conduction studies are consistent with an isolated complete injury of the common peroneal nerve. Which of the following deficits is most likely on physical examination?
A) Dorsiflexion of ankle
B) Plantarflexion of great toe
C) Sensation of lateral foot
D) Sensation of medial foot
E) Sensation of plantar foot

A

The correct response is Option A.
The common peroneal nerve forms as the sciatic nerve bifurcates at the apex of the popliteal fossa. It
then follows the medial border of the biceps femoris muscle and tendon. The nerve then passes over the
posterior aspect of the fibular head and winds around the neck of the fibula. The common peroneal then
divides into the deep and superficial peroneal nerve branches. The deep branch supplies the anterior
muscles of the leg, the dorsum of the foot, and the skin of the first web space. The superficial branch
supplies the peroneus longus and brevis muscles and the skin on the distal third of the lower leg and
dorsum of the foot. Because of its relatively superficial position, the common peroneal nerve is the most
commonly injured nerve of the lower extremity. Transection of the common peroneal nerve results in
paralysis of all muscles in the anterior and lateral compartments of the leg (dorsiflexors and ankle
evertors). This pattern of injury results in the classic picture of a foot drop. The distribution of sensory loss
would include the anterolateral leg and dorsum of the foot.
Sensation of the medial foot is from the saphenous nerve and branches of the medial plantar nerve. Lateral foot sensation is provided by the sural nerve. Sensation of the plantar aspect of the foot is from
the terminal branches of the tibial nerve (medial and lateral plantar nerves). All of the muscles of plantar
flexion of the ankle and toes (i.e. gastrocnemius, soleus, plantaris, and tibialis posterior, flexor hallucis
longus, flexor digitorum longus, and the intrinsic plantar foot muscles) are innervated by the tibial nerve.

How well did you know this?
1
Not at all
2
3
4
5
Perfectly
27
Q

A 26-year-old man sustained a crush injury to the tip of the left middle finger with an associated fracture at the dorsal base of the distal phalanx with nail bed injury 6 months ago. No treatment was provided.
Examination shows non-union of the distal phalanx. Which of the following is the most likely secondary deformity in this patient?
A) Boutonniere deformity
B) Jersey finger
C) Quadriga
D) Swan neck deformity
E) Trigger finger

A

The correct response is Option D.
The scenario described involves a bony mallet deformity in which a distal phalanx fracture is associated with disruption of terminal extension at the distal interphalangeal joint. If untreated, the DIP extension loss due to a non-union of a bony mallet injury may progress to a swan neck deformity through compensatory proximal phalangeal hyperextension in the setting of continued and persistent flexion at
the distal interphalangeal joint (from unopposed pull of the flexor digitorum profundus tendon). A secondary swan neck deformity may occur because of dorsal subluxation of the lateral bands and attenuation of the volar plate and transverse retinacular ligament at the PIP joint level.
A jersey finger is caused by rupture of the terminal flexor digitorum profundus. A boutonniere deformity can be caused by an injury to the central slip (but not the terminal extensor tendon). Quadriga is due to loss of length of a repaired FDP tendon, causing the finger with the repaired tendon to reach terminal flexion sooner than the other fingers whose FDP tendons are of normal length. A trigger finger does not
involve a fracture of the DIP joint.

28
Q

A 22-year-old man who is a college student sustains a volar oblique fingertip amputation while chopping
vegetables. Examination shows involvement of the hyponychium, but the nail is undamaged. The wound
measures 1 × 1.5 cm, and no exposed bone is noted. Which of the following is the most appropriate
treatment to encourage healing by secondary intention?
A) Apply negative pressure wound therapy
B) Apply povidone iodine to the wound daily and cover with dry gauze
C) Cover wound with semiocclusive dressing and change weekly
D) Leave wound open to air
E) Soak wound in hydrogen peroxide daily and cover with moist gauze

A

The correct response is Option C.
Fingertip or thumb tip amputations that result in small wounds (less than 1.5 cm 2) and minimal exposed bone are best managed with healing by secondary intention. The only exception to this might be a laborer anxious to get back to work with a healed wound sooner than 3 to 4 weeks. Mennen reported a series of 200 such injuries treated with a semi-occlusive dressing, and average healing time was 20
days.
A semiocclusive dressing is semi-permeable and transparent, allowing air to pass through the dressing, but providing a barrier to moisture. Commonly available semipermeable dressings are marketed under brand names like Tegaderm (3M) and OPSITE (Smith &Nephew). These dressings maintain a moist
wound environment, which speeds healing. If dressings are changed every 5 to 7 days, manipulation of
the wound is minimized and, therefore, healing is less disrupted.
Leaving a wound open to air would allow tissues to dry out, which would delay healing. Likewise, the use of povidone-iodine and/or hydrogen peroxide would slow down healing due to drying of the
wound. Although these topical agents are effective at eliminating bacteria from dirty or infected wounds,
prolonged use will interfere with normal wound healing. Finally, a wound of this small size would not
warrant negative pressure wound therapy. Even the small, intrinsically-powered negative pressure wound therapy devices would not offer any advantages over a semiocclusive dressing and would increase cost substantially.

29
Q

When a pedicled sural flap is raised to the heel, which of the following is the origin of the arterial blood
supply?

A) Descending genicular artery
B) Lateral sural artery
C) Medial femoral circumflex artery
D) Medial plantar artery
E) Peroneal artery

A

The correct response is Option E.
The reverse sural flap is a fasciocutaneous flap often used for ankle or heel wounds. The blood supply of the flap can be from a median superficial artery or the arterial plexus that travels with the sural nerve; the origin is a lower peroneal perforator located approximately 5 cm proximal to the lateral malleolus.
The lateral sural artery would be the appropriate blood supply for perfusion of a pedicled lateral
gastrocnemius flap. The gracilis flap blood supply derives from the medial circumflex artery. The
descending genicular artery provides the blood supply of the medial femoral condyle flap. The medial plantar artery is the blood supply for the medial plantar artery flap.

30
Q

A 37-year-old woman presents for evaluation of a laceration to the mid humerus that she sustained in a
motor vehicle collision. On examination, the patient is unable to extend the wrist, fingers, or thumb.
Surgical exploration shows complete radial nerve transection; the median/ulnar nerves are intact. Direct
neurorrhaphy is performed after debridement and mobilization of the nerve ends. Which of the following is the last muscle to be reinnervated during nerve recovery?
A) Abductor pollicis brevis
B) Abductor pollicis longus
C) Brachioradialis
D) Extensor carpi radialis brevis
E) Extensor indicis proprius

A

The correct response is Option E.
Radial nerve injuries may occur in the setting of humeral fractures, and transection is most common in the setting of an open injury. The most important components of functional recovery following radial nerve injury include wrist, finger, and thumb extension. The order of reinnervation of the radial-innervated muscles is most commonly brachioradialis, extensor carpi radialis longus, supinator, extensor carpi
radialis brevis, extensor digitorum communis, extensor carpi ulnaris, extensor digiti quinti, abductor
pollicis longus, extensor pollicis longus, extensor pollicis brevis, and extensor indicis proprius. The
abductor pollicis brevis is innervated by the median nerve via the thenar motor branch.

31
Q

A 37-year-old woman presents with sharp lateral elbow pain sustained when lifting a garbage bag out of
a can at work. Which of the following muscles is the most likely cause of this patient’s pain?
A) Anconeus
B) Brachioradialis
C) Extensor carpi radialis brevis
D) Extensor carpi radialis longus
E) Extensor carpi ulnaris

A

The correct response is Option C.
The common extensor tendon attaches the extensor carpi radialis brevis (ECRB), extensor digitorum
communis (EDC), extensor digiti minimi, and extensor carpi ulnaris to the lateral epicondyle. Among
these forearm extensor muscles, pathology found in the attachment of ECRB and EDC at the lateral
epicondyle (LE) is commonly cited as a reason for pain at the LE. While the extensor carpi radialis longus (ECRL) is a wrist extensor and may be a source of lateral elbow pain with strain, it does not originate from the lateral epicondyle. The ECRL muscle was observed to originate from the distal aspect of the supracondylar ridge mainly as a muscular attachment. The brachioradialis is an elbow flexor and originates off of the lateral column of the distal humerus. It is not involved with lateral epicondylitis. The
anconeus is a small muscle which originates off of the lateral epicondyle and its main function is to assist with elbow extension.

32
Q

A 28-year-old man sustains acute wrist extension during a fall on an outstretched arm. Examination
shows snuffbox tenderness. A scaphoid fracture is suspected. Which of the following imaging studies should be performed first to identify this patient’s injury?
A) Bone scan
B) CT scan
C) MRI
D) Plain x-ray studies
E) Ultrasonography

A

The correct response is Option D.
The correct answer is plain x-rays. Negative x-rays in scaphoid fractures are up to 30%. Cost
effectiveness of obtaining x-rays first is shown by the positive finding in 70%. The predictive value of clinical examination is 13-69% with an average of 21%. Depending on clinical suspicion and whether the patient needs to avoid immobilization if the absence of fracture can be confirmed, additional imaging
studies may be obtained.
For MRI, the estimated sensitivity is 97.7% and the specificity is 99.8% with 96% accuracy. For a
CT scan, estimated sensitivity is 85.2 to 94% and the specificity is 96 to 99.5% with 98% accuracy. Bone scintigraphy is 96 to 97.8% and 89 to 93.5%, respectively, with 93% accuracy. For follow-up x-ray
studies, 91.1 and 99.8%, respectively. MRI is therefore the best test for ruling in scaphoid fractures where the other tests are better at ruling out scaphoid fractures. Cost effectiveness of MRI for patients with suspicion for scaphoid fracture with negative x-rays is shown by getting patients out of unnecessary splints sooner.

33
Q

A 6-month-old male infant presents with bilateral radial longitudinal deficiency and thumb hypoplasia. Xray and photo are shown. Which of the following studies is most likely to rule out Fanconi anemia in this patient?

A) Bone marrow biopsy
B) Chromosomal breakage testing
C) Complete blood count with differential
D) Peripheral blood smear
E) Renal ultrasonography

A

The correct response is Option B.

All children with radial longitudinal deficiency (RLD) should undergo a thorough physical examination
because of the frequency of associated syndromes. Common conditions associated with RLD include thrombocytopenia-absent radius (TAR) syndrome, Holt-Oram syndrome, VACTERL syndrome and
Fanconi anemia.
Thrombocytopenia-absent radius (TAR) syndrome is an autosomal recessive disorder characterized by an absent radius with a relatively normal thumb. Holt-Oram syndrome is an autosomal-dominant disorder in which RLD is accompanied by either an atrial or ventricular cardiac septal defect. Fanconi anemia is an autosomal-recessive pancytopenia, once invariably fatal, that is now routinely treated with bone
marrow transplantation. One should consider further screening with spinal x-ray studies, cardiac
echocardiography, renal ultrasonography, and complete blood count.

Because children with Fanconi anemia often do not manifest aplastic anemia on routine blood testing until after age 3 years, it is also recommended that patients with RLD undergo a chromosomal breakage
test
for earlier detection of this potentially fatal disease. Routine genetic counseling is also
recommended. A peripheral blood smear and bone marrow biopsy would not be helpful in making this diagnosis.

34
Q

A 43-year-old man presents for reconstruction of a soft tissue deficit of the antecubital fossa with a reverse lateral arm pedicled flap. Which of the following arteries is the blood supply for this flap?
A) Anterior interosseous
B) Persistent median
C) Posterior interosseous
D) Radial recurrent
E) Ulnar

A

The correct response is Option D.

Although the lateral arm flap has predominantly been used in free tissue transfer for distant defects
based on the posterior radial collateral artery, transfer as a pedicled reverse-flow flap based on the radial recurrent artery has been both anatomically and clinically proven. Occasionally, it is performed with a
delay procedure at an intermediate stage.
The anterior and posterior interosseous arteries can provide circulation to perforator flaps. The ulnar artery has been occasionally used for an ulnar artery based fasciocutaneous flap. The persistent median
artery passes through the carpal tunnel and runs with the median nerve.

35
Q

A 19-year-old man is brought to the emergency department because of an injury to the right heel sustained during a lawn mower accident. After serial debridement is performed, there is a 2 x 2-cm soft
tissue defect with exposed calcaneus. Which of the following innervated flaps is most appropriate for
coverage of this defect?

A) Anterior lateral thigh flap with anterior femoral cutaneous nerve
B) Medial plantar artery flap with division of posterior tibial nerve
C) Radial forearm flap with superficial branch of the radial nerve
D) Reverse sural artery flap with saphenous nerve
E) Ulnar forearm flap with deep branch of ulnar nerve

A

The correct response is Option B.
Heel reconstruction is a difficult surgical problem with limited local options, relatively poor vascularity in the region, and weight-bearing requirements. Flap options include a variety of local flaps including
transposition or rotation flaps, fasciocutaneous flaps (e.g., medial plantar); local muscle flaps (e.g.,
abductor hallucis, flexor digitorum brevis, and abductor digiti minimi); reversed fasciocutaneous flaps
(e.g., sural); and free flaps. Although innervated (and therefore potentially sensate) free flaps can be performed, these are less predictable than local options. From the above answer choices, the best option is a local flap based on the medial plantar artery, which had sensation from the medial plantar nerve, a
branch of a division of the posterior tibial nerve. Another advantage of the medial plantar artery flap is that it covers the heel with glabrous skin, which may better be able to withstand weight-bearing. The other options are not correctly matched with flaps and nerves. Correct pairings of flap and cutaneous innervation are:

Anterior lateral thigh flap - lateral femoral cutaneous nerve

Ulnar forearm flap - medial antebrachial cutaneous nerve

Radial forearm flap - lateral antebrachial cutaneous nerve

Reverse sural artery flap - sural nerve

36
Q

A 35-year-old woman presents for examination of a mass of the right volar radial wrist. The patient
reports that the mass spontaneously appeared 6 weeks ago and occasionally gets bigger or smaller. On physical examination, the mass transilluminates. Which of the following joints is the most likely origin point of the mass?
A) Lunotriquetral
B) Metacarpotrapezial
C) Radioscaphoid
D) Scapholunate
E) Scaphotrapezial

A

The correct response is Option C.
Ganglia are benign soft tissue tumors that are most commonly encountered in the wrist but may occur in any joint.
Sixty to 70% of ganglion cysts are found in the dorsal aspect of the wrist and communicate with the joint via a pedicle. This pedicle usually originates at the scapholunate ligament but may also arise from a number of other sites over the dorsal aspect of the wrist capsule.
Thirteen to 20% of ganglion cysts are found on the volar aspect of the wrist, arising via a pedicle from the radioscaphoid, scapholunate interval, scaphotrapezial joint, or metacarpotrapezial joint, in that order of frequency. Neither dorsal nor volar ganglion cysts typically originate from the lunotriquetral ligament.

37
Q

A 65-year-old man presents after sustaining an injury to the dominant index finger from a table saw accident 4 weeks ago. The patient did not seek medical attention at the time of injury because of a lack of health insurance. Medical history includes poorly controlled type 2 diabetes mellitus. He smokes two
packs of cigarettes daily. Examination shows a 1-cm wound that appears to be down to the tendon, and the tendon sheath is visible at the edges of the wound. He is unable to actively flex the proximal or distal interphalangeal joints, but full passive mobility of the joints is noted. The patient has normal two-point discrimination on the ulnar side of the finger but diminished sensibility on the radial side; there is good capillary refill. Single-stage, flexor reconstruction with tendon graft is CONTRAINDICATED because of
which of the following clinical findings in this patient?
A) Absence of ipsilateral palmaris longus tendon
B) Injury to both flexor digitorum profundus and superficialis
C) Medical comorbidities
D) Radial digital nerve injury
E) Soft tissue injury over the proximal interphalangeal joint

A

The correct response is Option E.
Flexor tendon reconstruction with tendon grafting is a complex operation that also involves a significant postoperative therapy commitment from the patient. Early treatment of flexor tendon injuries decreases the need for grafting. With delayed presentation (usually greater than 3 weeks), the cut ends of tendon
degenerate and muscles shorten under myostatic contraction, and it is difficult or impossible to perform a primary repair of flexor tendon injuries. Therefore, in delayed presentations, flexor tendon reconstruction may require the use of tendon grafting, which is done in either single or multiple stages depending on the
need for reconstruction of pulley sheaths or other ancillary procedures like release of contracted joints. There are many relative contraindications to perform a reconstruction with a tendon graft, and these should all be considered together in determining appropriate candidates for these operations. Any patient who will not participate in postoperative therapy is not a candidate for reconstruction, either single
or multiple stage. Patients without adequate soft tissue coverage are not candidates for single stage
reconstruction, and this is the contraindication for single-stage reconstruction in this patient.
Successful reconstruction with tendon grafting requires early postoperative mobilization, while soft tissue coverage options (e.g., cross finger flap) require a period of immobilization that will result in significant adhesion formation and a poor outcome. Although an insensate digit is a relative contraindication for salvage, repair of the digital nerves in this patient may improve that variable, and this is not the best answer. Injury to both flexor tendons is not a contraindication to reconstruction. Medical comorbidities are
considerations to improve outcomes, but are not considered absolute contraindications. Several tendon graft options are available including the palmaris longus tendon, however, absence of this tendon is not a contraindication to reconstruction. Passive immobility of interphalangeal joints is another contraindication to flexor reconstruction and joint release should be done beforehand, followed by a period of hand
therapy to maintain joint mobility.

38
Q

A 35-year-old man presents 3 months after sustaining a Zone II flexor tendon injury of the middle finger.
The patient has been compliant with therapy since the early postoperative period. Examination shows the finger is mildly edematous. A 30-degree-flexion contracture at the proximal interphalangeal (PIP) joint is
noted, and active and passive motion is 30 to 50 degrees. Which of the following is the most appropriate next step in management of this patient?
A) Continued hand therapy
B) Contracture release
C) Flexor tenolysis
D) Tendon transfer
E) Two-stage flexor tendon reconstruction

A

The correct response is Option A.
In this scenario, the patient has developed joint stiffness and tendon adhesions following a Zone II flexor tendon injury and surgical repair. The most appropriate next step is to continue therapy until passive and active range of motion is optimized. The timing until maximal improvement or until one may consider secondary tendon surgery is controversial and recommendations range from 3 to 12 months. The timing
is usually individualized to each patient. Flexor tenolysis has been shown to be a useful and successful surgery for restoring motion after flexor tendon repair. The indications for this procedure are generally as follows: adequate soft tissue coverage, resolution of edema, passive motion greater than active motion, adequate joint motion, adequate muscle strength, and a compliant patient. In this scenario, the patient has equivalent active and passive motion as well as poor joint motion overall. In this case, the joint contracture is likely in part related to flexor tendon adhesions, and the indications for secondary surgery would not change. Contracture release by itself would not improve finger motion and may be contraindicated as anything that would cause additional scarring or barriers to tendon gliding should be
avoided. The presence of edema in this patient also indicates that the finger has not reached maximal
improvement with therapy. Flexor tenolysis should be considered a substantial surgery with inherent
risks, and all patients should be consented for a two-stage flexor tendon reconstruction at the time of surgery. However, one would not advocate a staged reconstruction if the primary repair was felt to be intact clinically and salvageable, as in this case. Tendon transfers may be considered for a tendon
reconstruction in certain instances, but this patient does not meet these criteria at this time.

39
Q

A 40-year-old man sustained traumatic amputation of all fingers of the dominant hand 3 months ago. Tripod pinch reconstruction is planned with a double second toe transfer. Which of the following arteries is most likely to be the dominant blood supply to the second toe transfer in this patient?
A) First dorsal metatarsal artery
B) First plantar metatarsal artery
C) Lateral plantar artery
D) Medial plantar artery
E) Third plantar metatarsal artery

A

The correct response is Option A.
The first dorsal metatarsal artery (FDMA) is the dominant blood supply (to the great toe and second toe) in approximately 70% of cases. The first plantar metatarsal artery (FPMA) is the dominant blood supply in
20% of cases. The FDMA and the FPMA have a similar vessel caliber in the remaining 10% of cases.
The dominant vascular pattern can be evaluated by careful retrograde dissection that begins at the dorsal
aspect of the first web space. The junction of the lateral digital artery of the great toe and the medial
digital artery of the second toe can be identified just above the intermetatarsal ligament. Proximal
dissection continues dorsally and plantarly to evaluate the FDMA and FPMA.
If the FDMA is the larger caliber vessel or of similar caliber to the FPMA, then the toe transfer can be
based on the FDMA. Proximal dissection of the FDMA to obtain length is relatively straightforward. In the setting of a plantar dominance, dissection of the FPMA is carried out proximally, which can be more challenging. Plantar proximal dissection is typically limited to the mid metatarsal level to avoid additional morbidity. If additional length is required on the FPMA pedicle, a vein graft can be used. It is important to
note that in bilateral second toe transfers, the dominant vascular pattern can be asymmetric in 20% of patients.

40
Q

A 66-year-old man presents with a mass along the ulnar margin of the index finger proximal phalanx. The
patient reports that over the past year, the mass has grown in size. Physical examination shows the
mass is now interfering with digital flexion. X-ray studies show bony erosion into the cortex of the middle
phalanx. Excisional biopsy demonstrates a tan, multilobulated mass that has surrounded the digital nerve
and invaded the bone cortex. Which of the following is the most likely diagnosis in this patient?

A) Chondrosarcoma
B) Epidermal inclusion cyst
C) Giant cell tumor of tendon sheath
D) Retinacular cyst
E) Schwannoma

A

The correct response is Option C.
Giant cell tumors of the tendon sheath are the second most common hand masses. They are slowgrowing
and painless, and affect the volar surfaces of the index, middle and ring finger, at the PIP or DIP
joints. They usually appear tan or yellow, lobulated, and well-circumscribed. Bony erosion secondary to
long-standing pressure may be observed on radiographs. Treatment is excision, with recurrence rates
ranging from 0 to 44 percent. High recurrence rates are associated with satellite lesions, poor
encapsulation, distal locations, intraosseous involvement, concurrent degenerative joint disease, or
involvement of the adjacent joint/tendon. Radiotherapy following surgical excision has shown recurrence
rates as low as 4 percent.
Schwannomas are benign peripheral nerve tumors derived from Schwann cells, that can involve the
nerves of the hand. They are well encapsulated and slow growing, and typically arise over flexor
surfaces. They present as a soft, nontender mass that is mobile and may cause associated paresthesias.
Magnetic resonance imaging may be helpful to evaluate for malignant characteristics. These tumors can
often be shelled out because they involve the nerve sheath instead of individual fascicles, with little risk of
postoperative neurologic deficits. Malignant transformation is rare. Epidermal inclusion cysts are
painless, firm, keratin-filled cysts developing from traumatic implantation of epithelium into the
subcutaneous tissue. Typical locations include the volar palm and digits. No risk of malignant
transformation has been reported, but cortical destruction can be observed, raising suspicion for a
neoplastic process. There is a low rate of recurrence with surgical excision. These masses can be
differentiated from giant cell tumors of the tendon sheath, in that they are cystic structures filled with
keratin and not mutilobulated, solid masses.
Ganglions are usually solitary and occur in specific locations in the hand and wrist. The most common
ganglion locations are the dorsal and volar wrist regions. They also occur in the digital flexor tendon
sheath (retinacular cyst), arising from the A1 pulley. There is no consensus regarding the preferred
Test Review Report
Printed on: 2/26/2023
treatment of a flexor tendon sheath ganglion. Historically, the large number of therapeutic options
described suggests that a predictable treatment approach could not be agreed upon. Recent literature
indicates that there are two acceptable treatment options: cyst aspiration or surgical excision.
Chondrosarcomas are the most common primary malignant bone tumors of the hand, most frequently
affecting the proximal phalanx. They may arise de novo or from malignant transformation of benign
cartilaginous lesions such as enchondromas. At imaging, it may often be difficult to distinguish
chondrosarcomas from their benign chondroid counterparts such as enchondromas. Phalangeal
chondrosarcomas are locally aggressive and, unlike chondrosarcomas of other skeletal structures, rarely
metastasize.
Cortical breakthrough, irregular cortical thickening, and a soft-tissue component are all suggestive of
chondrosarcoma rather than benign tumors such as enchondroma. Recurrence rates with intralesional
excision are historically high, such that wide excision or amputation had previously been the mainstay of
surgical treatment. Recent literature, however, advocates intralesional excision with close follow-up for
low-grade lesions or phalangeal tumors in circumstances where amputation will result in significant
functional loss. No role for irradiation or chemotherapy has been described.

41
Q

A healthy 11-year-old boy is brought to the emergency department because of worsening redness and pain around the site of an injury to the middle finger of the left hand. The patient sustained the original injury 3 weeks ago while wrestling with his brother. The patient had swelling and pain of the finger, but
the pain resolved quickly, so no medical care was sought. The patient’s mother reports that she noticed a small bump on the dorsum of the finger since then. However, the patient developed redness and pain 2 days ago at the same site, both of which have worsened. An image and x-ray studies are shown. Which
of the following is the most likely cause of this patient’s symptoms?

A) Biting of nails
B) Exposure to Pasteurella species
C) Flexor tendon avulsion
D) Immunocompromised state
E) Trapped germinal matrix

A

The correct response is Option E.
This patient has a Seymour fracture—a juxta-epiphyseal open fracture—of the distal phalanx as
evidenced by the eponychial disruption and fracture pattern on x-ray study. These are open fractures
traditionally frequently with associated nail bed transection. Unfortunately, secondary to delay in
treatment, the patient developed osteomyelitis.
These patients often present with a mallet-appearing deformity from flexion of the distal fracture segment,
the nail may be disrupted (with the distal nail bed) and lay superficial to the eponychial fold, while the
transected nail bed proximally (germinal matrix) becomes entrapped within the fracture, making closed reduction prone to failure. Acute treatment in the emergency department or operating room consists of nail plate removal, reduction of the trapped nail bed, irrigation and debridement of the fracture site, reduction of the fracture, nail bed repair, nail plate replacement, and immobilization. Instability or inadequate reduction warrants operative intervention and may require Kirschner wire fixation. Inadequate reduction and/or delayed treatment are critical influences of infection rates. In a study by Reyes and Ho in the Journal of Pediatric Orthopaedics, investigators reviewed acute appropriate reduction, acute partial treatment, and delayed treatment. No infections occurred in the acute appropriately managed group, whereas 15% of the patients developed infections in the partially treated group, and 45% of the delayed
treatment group developed infections. Biting of nails has been associated with nontraumatic paronychial infections. There is no history of immunocompromised state, and healthy children can acquire infections with these injuries. This patient sustained the injury wrestling his brother as opposed to from an animal
bite. Pasteurella is not the most frequent bacteria associated with infections in patients who have
sustained Seymour fractures. Flexor tendon avulsions, also known as jersey fingers, are traditionally closed and would demonstrate lack of flexion of the distal interphalangeal joint.

42
Q

A 50-year-old woman previously diagnosed with left forearm compression neuropathy of the superficial
radial nerve comes to the office for examination. The patient has not responded to 7 months of
conservative management which consisted of NSAID therapy, steroid injection, a trial of splinting, and
activity modification. Operative treatment is planned. Fascial release between which of the following two
tendons is most appropriate in this patient?
A) Abductor pollicis longus and extensor pollicis brevis
B) Brachioradialis and extensor carpi radialis longus
C) Extensor carpi radialis longus and extensor carpi radialis brevis
D) Flexor carpi radialis and abductor pollicis longus
E) Flexor carpi radialis and brachioradialis

A

The correct response is Option B.
Superficial radial nerve compression of the forearm occurs most frequently at the posterior border of the
brachioradialis where the nerve transitions from a deeper, subfascial position to a more superficial, subcutaneous location. Also known as Wartenberg syndrome, patients may present with pain, numbness, or tingling over the dorsal radial hand radiating to the dorsal thumb and index finger.
Symptoms of superficial radial nerve compression may be confused with symptoms of de Quervain’s tenosynovitis. In addition, both conditions may coexist simultaneously.
Patients diagnosed with superficial radial nerve compression are initially treated conservatively since this approach is successful in relieving symptoms in the majority of cases. Conservative management consists of rest, splinting, removal of external compression source (such as a tight wristwatch band,
bracelet, or handcuffs), and nonsteroidal anti-inflammatory medications. Surgery is indicated when conservative measures fail. Surgical decompression involves release of the fascia between the brachioradialis and extensor carpi radialis longus tendons. It is at this interval that the nerve transitions from deep to superficial and prone to compression. The other responses do not reflect the correct surgical anatomy of this condition.

43
Q

A 29-year-old man presents with nail pitting, leukonychia, and crumbling of the nail plate. A photograph is
shown. Which of the following is the most likely diagnosis in this patient?

A) Arsenic toxicity
B) Human immunodeficiency virus
C) Lichen planus
D) Psoriatic arthritis
E) Subungual melanoma

A

The correct response is Option D.
Psoriatic arthritis often presents with auto-fusion of the small hand joints and diffuse fusiform swelling of
the digits. This psoriatic dactylitis or “sausage digit” is caused by inflammation of periosteum, tendon, and
tendon insertions. Nail deformities include pitting, leukonychia, nail crumbling, and onycholysis
(separation of nail plate from bed). Nail deformities affect approximately 80% of patients with psoriatic
arthritis, and 50% of patients with psoriasis.

Nail lichen planus (NLP) is characterized by thinning, longitudinal ridging, and distal splitting of the nail
plate. NLP is usually resistant to topical corticosteroid therapy, but successful treatment has been
reported with intralesional or systemic administration of corticosteroids.
Characteristic skin lesions of arsenic poisoning include hyperkeratosis and Mees’ lines. Mees’ lines are
prominent transverse white lines in fingernails or toe-nails due to arsenic deposition in keratin-rich
tissues.
Subungual melanoma has distinct cutaneous nail manifestations such as brown or black streaks in the nail without any known injury, streaks on the nails that increase in size, or a “bruise” on the nail that will not heal or move up as the fingernail grows. One of the key indications of subungual melanoma is known
as “Hutchinson’s sign.” This is when a person has a streak that extends from the tip of the nail down to the nail bed and into the eponychium.
Nail disorders in HIV-infected patients include clubbing, splitting of the nails, or discoloration (black or brown lines going either vertically or horizontally).

44
Q

A 21-year-old woman presents for follow-up evaluation of an injury to the right middle finger that occurred
when a leash wrapped around her finger while she was walking her dog. X-ray studies were negative for
fracture, but there was a volar dislocation of the proximal interphalangeal (PIP) joint. Despite reduction of
the dislocation, the patient continues to have inability to fully extend the finger, and a boutonniere
deformity has developed. Splint treatment is planned. Which of the following is the most appropriate
position for splinting?
A) Distal interphalangeal (DIP) joint extended, PIP joint extended
B) DIP joint flexed, PIP joint flexed
C) DIP joint flexed, PIP joint free
D) DIP joint free, PIP joint extended
E) DIP joint free, PIP joint flexed

A

The correct response is Option D.
This patient has sustained a central slip disruption of the right middle finger after volar proximal
interphalangeal (PIP) joint dislocation, resulting in an acute boutonniere injury. Volar dislocations at the
PIP joint are rare and can be irreducible if there is interposition of the lateral band or central slip in the
joint. It is important to test for range of motion following reduction and check for stability of the joint. In
cases of significant soft tissue injury and instability, the patient may show inability to extend at the PIP
joint, and the finger can take on an acute boutonniere deformity. Boutonniere deformities demonstrate
flexion at the PIP joint and hyperextension at the distal interphalangeal (DIP) joint. The extensor tendon
(central slip) is detached from its insertion at the base of the middle phalanx, with decreased ability to
extend the finger at the PIP joint. As a result, the lateral bands fall volar to the axis of rotation at the PIP
joint, changing their force from extension to flexion at the PIP joint. The extensor force of the lateral
bands is directed toward the terminal tendon at the DIP joint, resulting in hyperextension of the DIP
joint. If missed, this injury can progress to fixed contracture.
By splinting the PIP joint in extension and leaving the DIP joint free to flex, the central slip is held in place and allowed to heal back to the middle phalanx. Flexion of the DIP joint through movement will prevent
volar migration of the lateral bands and will tend to cause them to migrate dorsally, reversing the effect of
the boutonniere deformity. When the lateral bands move dorsal to the axis of rotation at the PIP joint,
their extensor force is restored at the PIP, and the DIP joint hyperextension resolves.
Splinting both DIP and PIP joints in extension will not encourage the dorsal migration of the lateral bands.
Splinting of the DIP joint in extension with the PIP joint free is performed in zone 1 extensor tendon
injuries (mallet finger). Splinting with the PIP joint in flexion will not reverse the extensor tendon lag, and
will allow the deformity to persist. This position may be used in unstable dorsal dislocations to prevent
redislocation (extension block splinting).

45
Q

A 45-year-old patient has a slowly growing mass along the flexor sheath of the index finger at the level of the distal interphalangeal joint. The mass does not transilluminate and appears multinodular. It shows
generally decreased signal intensity on both T1- and T2-weighted MRI. Which of the following surgical procedures is most likely to be recommended?

A) Distal interphalangeal level amputation and lymph node biopsy
B) Incisional biopsy
C) Marginal excision
D) Radical excision

A

The patient most likely has a giant cell tumor of the tendon sheath. This is a benign nodular tumor that is found on the tendon sheath of the hands. It is also known as pigmented villonodular tumor of the tendon sheath. It is the second most common soft-tissue tumor seen in the hand, following ganglion cyst. There are no known cases of metastasis of this tumor in the literature. The operative treatment is marginal excision, and literature reports a 5 to 50% recurrence rate, more common if the tumor extends into the
joints and deep to the volar plate. Local recurrence is usually treated by repeat excision.
Incisional biopsy does not remove the entire tumor and is usually done for diagnosis, not treatment. A radical excision and amputation removes normal structures surrounding the tumor and will lead to unnecessary loss of function.

46
Q

Surgical reconstruction of the oblique retinacular ligament is primarily used to treat which of the following?
A) Boutonnière deformity
B) Flexor digitorum profundus tendon avulsion
C) Sagittal band rupture
D) Scapholunate ligament injury
E) Swan-neck deformity

A

The correct response is Option E.

Spiral oblique retinacular ligament (SORL) reconstruction is used to address digital swan-neck deformity.
The procedure is predicated on surgically recreating the proposed function of the oblique retinacular
ligament (ORL); linking proximal interphalangeal (PIP) joint and distal interphalangeal (DIP) joint flexion and extension. The ORL arises from the flexor tendon sheath at the base of the proximal phalanx, extends distally and dorsally over the PIP joint, and fuses with the lateral extensor tendon. Because of the site of the ORL origin and insertion, PIP joint extension places the ORL under tension, which results
in DIP joint extension. PIP joint flexion, however, decreases tension in the ORL and allows DIP joint flexion. Although anatomical studies are conflicting regarding the actual presence and function of the ORL, surgically recreating this proposed function has been used to treat swan-neck and mallet deformities. Thompson, Littler, and Upton described the dynamic SORL reconstruction in 1978 as an alternative to static techniques, such as transferring a single lateral band volarly to prevent PIP joint hyperextension. In a SORL reconstruction, a free tendon graft is fixated to the dorsal distal phalanx,
passed over the radial aspect of the middle phalanx, and then “spiralled” palmarly across the PIP joint, where it is then secured to the ulnar aspect of the proximal phalanx.
A flexor digitorum profundus avulsion (i.e. Jersey finger) is typically managed by reinserting the tendon into the volar base of the distal phalanx. If delayed greater than 8 to12 weeks, interposition tendon
reconstruction may be necessary. A boutonniere deformity generally occurs as a result of a central slip
injury; this leads to a flexed posture of the PIP joint and an eventual hyperextended posture of the DIP joint as the lateral bands translate volarly. If closed, splinting is indicated. If the central slip was lacerated, an open repair is recommended. A sagittal band tear may cause pain and subluxation of the extensor
mechanism at the MCP joint and can be treated with splinting or repair. A scapholunate ligament tear can be directly repaired if acute, or reconstructed using a variety of techniques in the absence of scapholunate advanced collapse and arthritis of the wrist. If arthritis has developed, then partial or total
wrist fusion may be indicated.

47
Q

A 16-year-old boy develops a severe left first web space contracture 8 months after undergoing skin
grafting for a soft-tissue avulsion injury. At the time of contracture release, a pedicled fasciocutaneous
flap is planned for coverage of the soft-tissue defect. On the basis of the preoperative markings for the
flap in the photographs shown, the flap pedicle is located between which of the following muscles?

A) Brachioradialis and extensor carpi radialis longus
B) Brachioradialis and flexor carpi radialis
C) Extensor digiti minimi and extensor carpi ulnaris
D) Extensor digiti minimi and the extensor digitorum communis
E) Extensor digitorum communis and extensor carpi radialis brevis

A

The correct response is Option C.

The photograph illustrates the markings for a reverse posterior interosseous artery (PIA) flap. The
reverse PIA flap is a thin, pliable fasciocutaneous flap that can provide reliable coverage of soft-tissue defects involving the dorsal hand, metacarpophalangeal joints, and first web space. Some surgeons report success using this flap for coverage of palmar wounds and soft-tissue injuries of the thumb as well.
Perfusion of the flap is based on retrograde flow through the posterior interosseous artery, which sends septocutaneous perforators to the overlying skin. The axis of the flap can be marked corresponding to a line between the lateral epicondyle and the radial aspect of the ulnar styloid. The location of the posterior interosseous artery pedicle is between the extensor digiti minimi and the extensor carpi ulnaris.
Retrograde perfusion through the flap relies on an intact communication of the PIA with the dorsal branch of the anterior interosseous artery, which is present in nearly all cases. This anastomosis is located 2 cm proximal to the radial aspect of the ulnar styloid; therefore, it corresponds to the pivot point of the flap.
One of the advantages of this flap is that it does not require sacrifice of a major arterial source of blood to the hand.
The other responses do not correctly describe the location of the PIA. Of note, the interval between the brachioradialis and the flexor carpi radialis represents the location of the radial artery fasciocutaneous flap pedicle.

48
Q

During flexor tendon repair, which of the following is the optimal distance from the cut end of the tendon for placement of core sutures?
A) 1 to 2 mm
B) 3 to 4 mm
C) 7 to 10 mm
D) Greater than 15 mm

A

The correct response is Option C.

The fundamentals of flexor tendon repair are based on primary tendon repair principles, which include easy placement of sutures in the tendon, secure suture knots, smooth juncture of the tendon ends, minimal gapping at the repair site, minimal interference with tendon vascularity, and sufficient strength throughout healing to permit application of early motion stress to the tendon.
These fundamentals are best achieved by incorporating a few basic principles. Handling of the tendon should be minimized to decrease the occurrence of adhesion formation. The strength of the repair is proportional to the number of core sutures and the caliber of the sutures that cross the repair site. The core sutures should be placed 7 to 10 mm from the tendon edge; dorsal placement is biomechanically
advantageous.
The distance of the tendon-suture junctions relative to the level of the tendon cut affects the strength of repairs of cut tendons. Strength of repair decreases significantly with purchase distance of less than 4 mm. No increase in strength is seen with purchase distances of greater than 7 mm, and attempts to increase the purchase distance more significantly (greater than 15 mm) will potentially require
unnecessary pulley disruption to achieve and will predispose to bunching at the repair site.

49
Q

Which of the following is the primary advantage of repairing a nail bed laceration with 2-octyl
cyanoacrylate compared with suturing?
A) Better cosmetic outcome
B) Better functionality
C) Less pain
D) Shorter repair time

A

The correct response is Option D.
Nail bed repair can be performed using suture or with 2-octylcyanoacrylate. In a study by Edwards and Parkinson, functional outcomes were equivalent between the two techniques, but 2-octylcyanoacrylate repair was significantly faster.

50
Q

A 29-year-old man is evaluated 15 months after a motorcycle collision in which he sustained a left humerus fracture and a complete left brachial plexus avulsion. He has not recovered any motor or sensory function of the left arm but desires the ability to actively flex the elbow. Which of the following procedures is most likely to restore active elbow flexion in this patient?
A) Free functioning gracilis muscle transfer
B) Pedicled bipolar latissimus dorsi muscle transfer
C) Proximal transfer of the flexor-pronator mass onto the humerus
D) Triceps-to-biceps transfer
E) Ulnar nerve to musculocutaneous nerve transfer

A

The correct response is Option A.
Adult traumatic brachial plexus injuries are devastating and life-altering injuries. Evaluation includes a detailed physical examination and radiologic and electrodiagnostic studies. Knowledge of injury patterns,
timing of surgery, prioritization in restoration of function, and management of patient expectations are key
components of management. In general, options for treatment of brachial plexus injuries include
neurolysis, nerve grafting, or nerve transfers and should be performed within 6 months of injury. Free functioning muscle transfers (FFMT) and tendon transfers are typically used in patients who present late
(greater than 12 months from injury), because the time for the nerve to regenerate after reconstruction is greater than the survival time of the motor end plates after denervation.
FFMT is the transplantation of a muscle and its neurovascular pedicle to a new location to assume a new function. The muscle is innervated by transferring an intact uninjured donor motor nerve; circulation is
restored to the muscle through microsurgical anastomosis of the artery and vein to donor vessels
(typically the thoracoacromial artery and cephalic vein). Within 6 to 9 months, the transferred muscle reinnervates, eventually gaining independent function. Although initially indicated in patients who presented late or as a salvage procedure with failed previous nerve reconstruction, the success with FFMT has led to use in early reconstruction to obtain elbow flexion and rudimentary grasp in patients with
pan-plexus injuries. The gracilis muscle is the most commonly used donor because of its proximally based neurovascular pedicle and its long tendon length (which can reach distally into the forearm). Spinal accessory or intercostal nerves could be used as donor nerves for a free gracilis transfer. Steindler flexorplasty, pedicled latissimus dorsi muscle transfer, triceps-to-biceps transfer, and Oberlin transfer each require remaining function of the brachial plexus and would be contraindicated in this patient with
long-standing total brachial plexus palsy.

51
Q

A 44-year-old woman sustained a second-degree burn to the volar surface of the wrist and palm. She is sent to occupational therapy for fabrication of a splint, placing the wrist and fingers in an intrinsic plus position. Which of the following best describes the position of the wrist and fingers in this splint?
Wrist / Metacarpophalangeal Joint / Interphalangeal Joints
A) 30 degrees extension / 75 degrees flexion / 30 degrees flexion
B) 30 degrees extension / 0 degrees flexion / 0 degrees flexion
C) 30 degrees extension / 75 degrees flexion / 0 degrees flexion
D) 30 degrees flexion / 75 degrees flexion / 0 degrees flexion
E) 30 degrees flexion / 0 degrees flexion / 30 degrees flexion

A

The correct response is Option C.
The position of safe immobilization (POSI), also called the intrinsic plus position, was described initially by J.I.P. James (British orthopedic surgeon) and is recognized as the correct position in which to immobilize the hand safely following injury or surgery. The wrist is placed in 0 to 30 degrees of extension,
metacarpophalangeal (MCP) joints in 70 to 90 degrees of flexion and interphalangeal (IP) joints in full
extension. This position creates pretension on the collateral ligaments of the wrist and the MCP and IP joints of the hand, thereby decreasing the risk of stiffness and contracture.

52
Q

Which of the following is the most appropriate initial management of lower extremity lymphedema?
A) Charles operation
B) Compression
C) Liposuction
D) Lymph node transfer
E) Lymphatic venous bypass

A

The correct response is Option B.
Lymphedema is a chronic condition that creates significant dysfunction and disability. Causes can be developmental or the result of lymphatic injury, such as in oncologic procedures. Although there is no cure for the condition, surgical techniques have demonstrated promise. However, these should be reserved for patients who have failed conservative management with compression therapy, lifestyle
modification, and therapeutic exercise. Complete decongestive therapy consistently demonstrated effectiveness in limb volume reduction but requires ongoing treatment and compression to maintain its
benefits.

53
Q

Which of the following diagnostic findings is most consistent with a recovering motor nerve injury?
A) Decreased motor unit potential amplitude
B) Decreased motor unit recruitment
C) Fibrillation potentials
D) Nascent potentials
E) Positive sharp waves

A

The correct response is Option D.
Electrical studies of a recovering nerve injury would show nascent potentials. This finding usually precedes the onset of clinically evident voluntary movement in the muscles innervated by the injured
nerve. Nascent potentials appear several months after injury and result from axonal regeneration.
Decreased motor unit potential amplitude, fibrillation potentials, positive sharp waves, and decreased motor unit recruitment are possible diagnostic findings in the setting of a nerve injury, but they are not indicative of recovery. Nerve lesions that spontaneously recover are usually treated nonsurgically, whereas those without recovery are explored and reconstructed. As a general rule, nerve regrowth occurs at approximately 1 inch per month or 1 millimeter per day. Motor endplates degrade at about 1%
per week, hence the maximum length that a nerve can grow to restore motor function is approximately 13 to 18 inches. Repairs at the brachial plexus level rarely result in the recovery of any intrinsic muscle function. Sensory end organs, however, remain viable and can be reinnervated even after many years.

54
Q

A 32-year-old woman sustains a dorsal proximal interphalangeal (PIP) joint fracture-dislocation of the ring
finger after a fall from standing. The fracture-dislocation is reduced, and the finger is splinted in the
emergency department. Three days after injury, the patient presents to the office. An x-ray study demonstrates a volar lip fracture of the middle phalanx, which measures 40% of the joint. Live fluoroscopy demonstrates PIP joint congruency through an arc of motion of 20 to 100 degrees. Which of the following is the best treatment for this patient?

A) Dorsal block splint
B) Hemi-hamate resurfacing
C) Open reduction and internal fixation
D) PIP joint transarticular pinning
E) Volar static splint

A

The correct response is Option A.
Proximal interphalangeal (PIP) joint fracture-dislocations are generally stratified according to the percentage of middle phalanx articular surface disrupted: 30% or less, presumed stable; 30 to 50%,
tenuous stability; and more than 50%, unstable. Stable fracture-dislocations (less that 30% of the middle phalanx palmar lip) are stable in full PIP joint extension. When 30 to 50% of the middle phalanx joint surface is fractured, stability is tenuous and can only be determined by clinical testing. Those fractures that require more than 30 degrees of flexion to maintain reduction are considered unstable and therefore require some form of surgical management for maintenance of reduction (dorsal block pinning, or open reduction and internal fixation).
For all fracture-dislocations, treatment methods are grouped into five broad categories: static
immobilization, articular reduction, dorsal extension block splinting, open reduction and internal fixation, and volar plate arthroplasty.
Static immobilization is straightforward but generally avoided in cases of fractures involving the PIP joint,
owing to the risk of overwhelming stiffness with prolonged immobilization.
Articular reduction must be serially monitored by x-ray study, and immobilization for more than 3 weeks may result in significant PIP joint stiffness. Dorsal extension block dynamic splinting can be used for stable or tenuous fracture-dislocations.
Dorsal block pinning uses the same concept as dorsal block splinting, but it requires the surgical insertion of a pin into the head of the proximal phalanx to prevent dorsal translocation of the middle phalanx with extension.
Open reduction and internal fixation can be used to fix a noncomminuted fragment that is sufficiently large enough to stabilize with small lag screws or K-wires.
Malerich and Eaton describe a procedure called volar plate advancement (advancement of the volar plate into the middle phalanx fracture defect), simultaneously restoring stability and resurfacing the damaged articular surface. Hastings and Kiefhaber describe hemi-hamate resurfacing arthroplasty (a technique used to resurface the volar lip of the middle phalanx with a hemi-hamate bone graft). Both of
these resurfacing techniques (volar plate arthroplasty and hemi-hamate arthroplasty) are salvage
procedures, better suited for chronic or subacute unstable fracture dislocations.
The fracture in this scenario involves 40% of the joint and is thus considered tenuous, but it achieves stable reduction at 20 degrees of flexion, allowing for treatments such as dorsal block splinting or pinning.
Given a choice, nonoperative management is preferred initially, and surgical correction (dorsal block pin or open reduction and internal fixation) is reserved for those that fail nonoperative management.

55
Q

A 36-year-old man presents to the clinic 1 year after repair of an isolated brachial artery laceration. Prior to arterial repair, the hand and forearm were dysvascular. Fasciotomies were not performed at the time of repair. The patient is unable to extend his fingers actively or passively with the wrist held in neutral position, but he is able to actively make a full fist. Sensation is intact. Which of the following interventions is most appropriate to improve finger extension and preserve grip strength in this patient?

A) Flexor pronator slide
B) Free functional gracilis transfer
C) Joint release and tenolysis
D) Splinting

A

The correct response is Option A.
Volkmann ischemic contracture is a devastating condition with serious motor and sensory functional implications for the upper extremity, most typically the forearm. It is the result of an acute compartment syndrome, following severe soft-tissue trauma and accompanying vascular insult. The patient in the scenario demonstrates a moderate contracture that is best treated with a flexor pronator slide.
When treatment of acute compartment syndrome is delayed or neglected, the muscles of the forearm undergo necrosis and contracture due to secondary fibrosis, causing the typical flexed deformity. This results in impairment of hand and finger function.
Surgical treatment is based on severity of contracture and function of the residual motor units. Mild contractures allow for full passive extension of the fingers with the wrist in volar flexion and can be treated with tendon lengthening and skin release, or selective flexor pronator slide, depending on the
source of constrainment. Patients with moderate contractures demonstrate an inability to passively extend the fingers with the wrist in flexion but retain flexor muscle function. These contractures require consideration for a flexor pronator slide alone or in conjunction with tendon lengthening. Complete loss of
muscle function necessitates consideration of free functional muscle transfer. Superficialis to profundus transfers are a consideration in the setting of significant contracture and functional limitation. It is typically
used to facilitate improved hygiene and confers limited function. This would not be as good of an option for the patient in this question as it would compromise his strength and function. Neurolysis should be
considered in conjunction with any reconstructive procedures. Splinting is an important adjunct to any reconstructive procedure and potentially can be employed as an initial treatment prior to surgical intervention to prevent worsening contracture.

56
Q

A 19-year-old college baseball player sustained an injury to his left hand when hitting the ball with a bat 6
weeks ago. The patient reports immediate pain in his palm, weakness of grip, and within hours
developed tingling into his small and ring fingers, which persisted for 6 weeks despite rest. He reports
continued tingling in the ulnar fingers. Standard x-ray studies of the hand show no abnormalities. A CT
scan of this patient’s hand is likely to show a fracture of which carpal bone?
A) Capitate
B) Hamate
C) Lunate
D) Pisiform
E) Scaphoid

A

The correct response is Option B.
This college student has sustained a hook of hamate fracture that was missed on standard hand x-ray
studies. Impact from the bat or from catching a pitched ball are common mechanisms of injury for a hook
of hamate fracture in a baseball player. While fractures of other carpal bones may occur in baseball
players, they would not produce the symptom spectrum described in the scenario.
An x-ray study from the carpal tunnel view and a CT scan will likely show the correct diagnosis. At 6
weeks, this patient is unlikely to improve with casting alone. Surgically, he may be offered open reduction
and internal fixation versus resection of the bony fragment. During surgery, the flexor tendons to the
small finger should be evaluated as they can sustain attritional injury next to the fracture fragment. Nerve
irritation to the ulnar nerve (sensory and motor) usually resolves spontaneously after surgery, but the
nerve should be examined during surgery.

57
Q

A male newborn is evaluated in the newborn nursery because of limited movement of the right arm.
Maternal history includes gestational diabetes, but routine prenatal monitoring and ultrasound
examinations were normal. The patient’s delivery was difficult, and he weighed 10.5 lb (4.8 kg) at birth.
He was noted to have no spontaneous movement of the right arm. The right upper extremity was warm,
pink, and supple. Pulsations of the radial and ulnar arteries were palpable at the wrist. X-ray studies of the affected shoulder show no obvious fractures. A photograph is shown. Which of the following is the most appropriate next step in management?

A) Angiography of the extremity
B) Anticoagulation
C) CT scanning of the extremity
D) Measurement of compartment pressures
E) Observation

A

The correct response is Option E.
This patient has an obstetrical palsy of the right upper extremity. The likelihood of recovery depends on the severity of the injury, but statistically over 70% of patients have complete or near complete recovery of upper extremity function without any surgical intervention. Thus the most appropriate next step for this newborn is observation. MRI of the shoulder and neck are helpful to discern evidence of anatomical injury to the cervical roots and/or portions of the brachial plexus, but CT scanning is unnecessary and of little use in this regard. Angiography and anticoagulation would be appropriate steps if there was clinical evidence of subclavian or brachial artery thrombosis, but the physical examination does not support this diagnosis. Similarly, the examination is inconsistent with neonatal compartment syndrome, a rare condition that usually presents with arm swelling, immobility, reduced arm perfusion, and purple
cutaneous areas. Therefore, measurement of compartment pressures is unnecessary.

58
Q

A 43-year-old woman is evaluated because of lymphedema of the lower extremities. She demonstrates pitting edema, which does not improve with limb elevation. Her skin feels otherwise normal, with no
evidence of fibrotic change. Which of the following International Society of Lymphology stages best
describes this patient’s lymphedema?

A) 0
B) 1
C) 2
D) 3

A

The correct response is Option C.
Lymphedema results from congenital or acquired dysfunction of the lymphatic system. It results from
changes to the lymphatic vessels, including ectasia and valve dysfunction. This results in reflux of lymphatic fluid into the interstitial space. Lymphatic fluid accumulation leads to chronic inflammation, extracellular matrix remodeling and fibrosis, adipose tissue differentiation, progressive fibrosis/sclerosis, and eventual obliteration of the lymphatic vessel lumen. Over time, accumulation of interstitial lymphatic fluid causes subcutaneous fibroadipose production.
Lymphedema is a chronic condition that slowly worsens over time. It progresses through four stages.

Stage 0 indicates a clinically normal extremity but with abnormal lymph transport (identified via
lymphoscintigraphy).

Stage 1 demonstrates a relative accumulation of fluid high in protein content, which
improves with limb elevation. Pitting may occur.

Stage 2 represents pitting edema that does not resolve with elevation, but no evidence of fibrotic skin changes. Late in Stage 2, the limb may not pit as excess
fat and fibrosis begins.

Stage 3 describes fibroadipose deposition and fibrotic skin changes.
The presence of dermal backflow on contrast-enhanced imaging of the lymphatic system is diagnostic for lymphedema, and the severity and distribution of this backflow correlate closely with the pathologic condition of the lymphatic vessels. Indocyanine green fluorescent lymphography enables detailed dynamic functional evaluation of the superficial lymphatic system and can also be used for intraoperative
lymph node mapping for vascularized lymph node transplantation.

59
Q

A 16-year-old girl requires a skin graft to the dorsal forearm after she sustained injuries in a motorcycle accident. Which of the following wound coverage techniques will lead to the LEAST secondary contraction?

A) Full-thickness skin graft, 1:4 meshed
B) Full-thickness skin graft, nonmeshed
C) Healing by secondary intention
D) Split-thickness skin graft, nonmeshed, 0.010-in thick
E) Split-thickness skin graft, nonmeshed, 0.014-in thick

A

The correct response is Option B.
Secondary contraction of a wound depends on various factors. Studies have shown that thicker grafts result in decreased scar depth and secondary contraction, whereas thinner grafts contract more. In terms of numbers, thick split-thickness skin grafts (0.03 in, 760 mm) approach similar final wound sizes as full-thickness
skin grafts. Wider meshing results in more secondary contraction.
Myofibroblast differentiation is thought to be a key driver of contraction. Application of full-thickness skin grafts appears to speed up their life cycle, compared with wounds treated with split-thickness grafts.
Conversely, thin grafts show a greater expression of alpha-smooth muscle actin, which persists for a longer time after grafting. Other mechanisms include decreased expression of pro-inflammatory genes
one week after full-thickness grafting, such as insulin-like growth factor 1 (IGF1) and transforming growth
factor beta-1 (TGFB1), when compared with wounds treated with thin split-thickness grafts.
Healing by secondary intention is associated with greater contraction than use of a graft.

60
Q

A 10-year-old girl is brought to the office by her mother because of difficulty using her hand. Medical history includes supracondylar fracture 6 months ago treated with a closed reduction and casting. The
fingers of the affected hand are held in the intrinsic minus position. Volkmann ischemic contracture following the fracture is suspected. Which of the following muscles is LEAST likely to be affected by Volkmann contracture?

A) Brachioradialis
B) Flexor digitorum profundus
C) Flexor digitorum superficialis
D) Flexor pollicis longus
E) Pronator teres

A

The correct response is Option A.
Volkmann ischemic contracture results from forearm muscle shortening and fibrosis as a result of ischemia of forearm muscles during increased compartment pressures. Common reasons for increased compartment pressures include gunshot wounds and fractures, particularly supracondylar pediatric
fractures. The radial artery is superficially located, whereas the ulnar artery is deeply positioned,
traversing deep to the pronator teres muscles. The ulnar artery becomes the common interosseous artery, which divides immediately into anterior and posterior interosseous branches. The muscles dependent on this deep circulatory pattern are more likely to be affected by ischemia during increased compartment pressures. Flexor muscles commonly involved in this process are the flexor digitorum
superficialis, flexor digitorum profundus, flexor pollicis longus, and pronator teres. The brachioradialis is not typically affected due to its more superficial radial artery circulation. Patients with significant functional
loss may require surgical procedures such as a free gracilis functioning muscle transfer.

61
Q

A 5-year-old boy presents with deformity of the tip of the left long finger as shown in the photographs.
The tip of the finger was amputated one year ago, and the wound was repaired at a local emergency department. Which of the following is the most appropriate method of correcting this deformity?

A) Coverage with a hypothenar flap
B) Distraction lengthening of the distal phalanx
C) Free toe transfer
D) Nail bed grafting
E) Release and augmentation of hyponychium

A

The correct response is Option E.

This is a classic hook nail deformity and is caused by deficient bone support of the distal nail bed, soft tissue
contracture/deficiency of the tip of the finger, or both. The most appropriate way to correct the
deformity is release and shortening of the portion of the nail bed that has no underlying bone support, and augmentation or advancement of the distal soft tissue envelope. There are many correction methods described, including some that add both soft tissue and bone support of the overhanging distal nail bed.
Regardless of method chosen, the primary goal of correction is to release the tethered nail bed, ensure that it is supported by bone, and provide sufficient soft tissue coverage to negate tension on the distal nail bed. Nail bed grafting alone will not correct this deformity as this does not provide additional bone support for the nail bed. A hypothenar flap is too remote to supply tissue for this problem. Lengthening the distal phalanx using bone grafting or vascularized bone has been described, but the use of distraction osteogenesis is impractical. Because most of the finger remains in place, a free toe transfer is not indicated.
Composite grafting could conceptually address this issue and has been described, but the survival of the graft is not predictable and harvest of the graft leaves a deformity at the donor digit.

62
Q

A 32-year-old man sustains a burn to his hand, resulting in a first web space contracture. A four-flap Zplasty
is planned for release of the contracture. Which of the following best describes the flap
reconstruction for this defect? Flap Type / Blood Supply
A) Advancement / axial
B) Advancement / random
C) Rotation / axial
D) Rotation / random
E) Transposition / axial
F) Transposition / random

A

The correct response is Option F.

Random pattern flaps are generally classified based on the primary motion of the flap. There are three basic types of tissue movement: transposition, advancement, and rotation. Transposition flaps
incorporate noncontiguous skin into a defect by lifting the flap over normal skin for inset into a defect.

Advancement flaps recruit adjacent tissue to close a defect via soft-tissue movement in a linear direction.

Rotation flaps move adjacent tissue around an axis to close a defect, rotating soft tissue into the defect.

Advancement and rotation flaps recruit adjacent lax tissue and move in either a linear or arced motion (respectively) to fill the defect.

Transposition flaps recruit noncontiguous tissue, which is lifted over intact soft tissue and placed into the defect.

Random pattern flaps lack a defined named arterial vascular
supply. Because of their lack of an axial vascular supply, they are subject to dimensional restrictions. In general, they are designed to not exceed a length:width ratio of 2:1.

An axial pattern flap is a single
pedicled flap that has an anatomically named arterial blood supply running along its long axis. Because of the presence of a named arterial blood supply, axial pattern flaps are not subject to the length:width ratio restrictions that apply to random pattern flaps.

63
Q

A 56-year-old man is evaluated because of high radial nerve palsy 12 months after sustaining a gunshot wound to the upper arm with complete radial nerve transection. To restore wrist and digit extension, tendon transfers are considered. Which of the following transfers is most appropriate for this patient?
A) Brachioradialis to extensor indicis proprius
B) Flexor carpi radialis to extensor digitorum communis
C) Palmaris longus to extensor pollicis brevis
D) Pronator quadratus to extensor carpi radialis brevis

A

The correct response is Option B.

Tendon transfers for complete high radial nerve injuries are often performed within weeks after injury and allow restoration of wrist and digital extensor stabilization.

1) If present, the palmaris longus tendon is transferred to the extensor pollicis longus tendon to allow for thumb extension.

2) The flexor carpi radialis is transferred to the extensor digitorum communis for finger extension.

3) The pronator teres is transferred to the extensor carpi radialis brevis to add support for wrist extension.

The brachioradialis is not generally a good transfer in a high radial nerve palsy as it is typically weak. The extensor indicis proprius does not usually receive a tendon for transfer as the extensor digitorum communis will provide extension to all digits, including the index. The extensor pollicis brevis does not normally receive a tendon transfer since thumb MP joint extension (in addition to IP joint extension) is normally restored with transfer to the extensor pollicis longus tendon.
Pronator quadratus is not used for tendon transfers for wrist extension and cannot reach the extensor carpi radialis brevis.

64
Q

A 25-year-old man has an acute fracture of the third metacarpal on his dominant hand. An x-ray study shows a displaced oblique fracture with shortening and rotation. During open reduction, which of the following muscles is most likely attached to the fracture fragments?
A) Abductor digiti minimi
B) Extensor digiti minimi
C) Extensor indicis proprius
D) Interosseous
E) Lumbrical

A

The correct response is Option D.
The correct answer is the second dorsal interossei radially and third dorsal interossei ulnarly. There is
little variation in atomic origins of the interosseous muscles. All interossei originate from the
corresponding metacarpal shafts. The extensor indicis proprius and extensor digiti minimi muscle bellies are located in the forearm. The abductor digiti minimi muscle is attached to the 5th metacarpal.
The lumbrical originates on the flexor digitorum profundus tendon and not on the bone.

65
Q

A 45-year-old carpenter presents with a six-month history of an ulceration of the ring fingertip and pain at rest. Digital brachial index is 0.45, and angiography demonstrates occlusion of the ulnar artery. The patient has tried three months of calcium channel blockers and aspirin without relief. Which of the following is the most appropriate treatment for this patient?
A) Amputation of the fingertip
B) Chemical sympathectomy
C) Reconstruction of the ulnar artery
D) Stellate ganglion block
E) Surgical sympathectomy

A

The correct response is Option C.
Conservative treatment includes smoking cessation, calcium channel blockers, anticoagulation therapy, stellate ganglion block, and behavior modification. Nonoperative management is generally considered
first-line treatment, because most patients will have at least partial resolution of their symptoms. With that said, 70% of those treated nonoperatively had partial resolution of their symptoms, and only 12% had complete resolution. Of patients treated operatively, 42% had complete resolution of their symptoms and 42% had partial resolution.
For patients with evidence of more advanced disease such as digital ulceration, chronic resting pain, or conservative management failure, operative intervention may be considered. Preoperative noninvasive vascular studies can be used to determine which patients may require reconstruction versus simple
excision and ligation. Studies have suggested that a digital brachial index less than 0.7 indicates
reconstruction may be warranted. An index of less than 0.5 suggests critical ischemia, which may result
in tissue loss.
Surgical options fall into two basic groups: resection of the involved arterial segment with ligation, and
vascular reconstruction with or without interposed graft. Graft occlusion is reported in as high as 78% of patients. Despite a high percentage of occlusion, patients remained satisfied. Patients with occluded reconstructions did not experience worsening of symptoms in comparison with the patent reconstructions. Preoperative digital brachial index values, although informative as to the patient’s digital perfusion, do not mandate a particular operative intervention. The general treatment algorithm is to
perform surgery on patients who have failed on medical management and local treatment to heal any digital soft tissues. A decision on ligation versus reconstruction can be made with the assistance of information gathered by preoperative angiography and noninvasive vascular studies, as well as
intraoperative assessment of ulnar digital perfusion with temporary occlusion of the ulnar artery. Poor perfusion following temporary occlusion mandates reconstruction of the artery, whereas adequate perfusion, despite occlusion, can be treated with simple excision or ligation of the diseased ulnar artery segment.